Download Test 1

Survey
yes no Was this document useful for you?
   Thank you for your participation!

* Your assessment is very important for improving the workof artificial intelligence, which forms the content of this project

Document related concepts

Nerve guidance conduit wikipedia , lookup

Cell encapsulation wikipedia , lookup

Organ-on-a-chip wikipedia , lookup

Bio-MEMS wikipedia , lookup

Tissue engineering wikipedia , lookup

Index of HIV/AIDS-related articles wikipedia , lookup

Transcript
Chapter 1 (cross reference to Dr. Anderson’s lectures/outlines on Cell Injury I & II)
A. Definitions. Be able to recognize examples of these conditions and vignettes and in pathologic
images from text, lecture outlines, and case studies. Several of these conditions are discussed in
greater detail in later pages of the text.
Necrosis: irreversible cell injury, sequence of morphological changes that follow cell death in living
tissue. Occurs after a loss of blood supply or after an exposure to toxins. Characterized by cell swelling,
protein denaturation, and organelle breakdown. Leads to tissue dysfunction. Inflammatory response
initiated.
Apoptosis: programmed cell death, dead cells removed without much disruption to surrounding tissues.
Where unwanted cells (ex: embryogenesis) or pathological cells (ex: irreparable mutations) are eliminated.
Inflammatory response is not initiated. Cells shrink in size.
Hypertrophy: increase in the size of the cells due to increased workload (ex: myocardial cells increase in
size if have excessive HTN or aortic stenosis because of the increased work load)
Atrophy: decrease in cell size without change in # of cells (ex: casted broken leg---don’t use will see
atrophy, remember use it or lose it)
Infarction: irreversible injury after complete or prolonged occlusion, cells are dead (ex: myocardial
infarction aka “heart attack”), irreversible
Cachexia: weight loss after periods of prolonged starvation (ex: the setting of malignant tumors)
Hyperplasia: increase in the number of cells, can be physiologic or pathologic, associated with
hypertrophy at times (exception: renal epithelial cells can undergo hypertrophy but not hyperplasia).
Physiological hyperplasia: a) hormonally induced (ex: during breast development and during pregnancy
have breast and uterine development) or b) compensatory hyperplasia (occurs when tissue is removed or
diseased ex: partial liver resection: cells will proliferate to fill in space where liver tissue once resided).
Hyperplasia is also important in wound healing. Pathological hyperplasia: mainly due to excessive
hormonal stimulation or growth factor stimulation (ex: warts caused by the papillovirus, tumors)
Aplasia: incomplete or underdeveloped cells/organs
Hypoplasia: underdeveloped cells/organs
Metaplasia: reversible change in which one adult cell type (epith or mesenchymal) is replaced by another
adult cell type
Hypoxia: oxygen deficiency which interferes with aerobic oxidative respiration, common cause of cell
injury/death
Hypoxemia: low oxygenation of blood
Anorexia: self induced starvation, results in cachexia, anemia may result leading to hypoxic tissues and
hypoxemia
Ischemia: loss of blood supply due to impeded arterial flow (due to occlusion) or decreased venous
drainage
Phagocytosis: engulfment of bacteria via invagination of membrane of pathogen to make phagosome
which later fuses with lysosome to make phagolysosome. With this fusion, the pathogen is ingested via
heterophagy or autophagy.
Autophagy: intracellular organelles (not outside sources) and portions of cytosol are sequestered from the
cytoplasm in the autophagic vacuole form from the ribosome-free parts of the RER (prominent n cells
undergoing atrophy). The vacuole fuses with a primary lysosome to form a autophagolysosome. This will
completely digest proteins, carbs, but some lipids remain undigested.
Heterophagy: materials from the external environment are taken up through a process generically called
endocytosis; uptake of larger particular matter is phagocytosis; and uptake of smaller macromolecules is
denoted pinocytosis. Mainly limited to phagocytotic cells: PMNs and macrophages
1
B. (pg 4-8) Understand causes and biochemical mechanisms of cell injury. Be able to recognize
specific examples and what happens at the subcellular level
Causes:
1. O2 deprivation
Examples:
anemia, CO poisoning
Subcellular events:
* ↓ Oxidative phosph = ↓ ATP
Leads to mito and cell swelling increase intracell
Ca++ which activates endonucleases, proteases,
ATPases via activation of phospholipases
2. Chemical agents
glucose, salt, high partial
pressures of O2, asbestos,
drugs, insecticides
* leads to free radical formation
causing lipid peroxidation
3. Infectious agents
tapeworm, bacteria, viruses,
fungi, protozoa
*viruses take over nuclear
activity making its own DNA
4. Immunologic rxts
incidental or intended,
*increase mast cell degranul.
anaphylaxis, autoimmune dz immune complexes deposited
in blood vessel walls etc
5. Genetic defects
Down’s syndrome, sickle cell
6. Nutritional imbalances
*cells misshaped in sickle cell
protein insufficiency, vitamin *decreased cell function
deficiencies, DM, atherosclerosis
7. Physical agents
trauma, extreme temps, radiation, *decreased cell function
electric shock, sudden changes in
atmospheric pressure
8. Aging
repeated injury
*decreased cell function
**NOTE: there subcellular events are difficult to find**
C. (pg 8 Fig. 1-6) Understand the sequence of events in ischemic injury in the mitochondria
Ischemia
↓
Decreased Oxidative
Phosporylation
↓
Decreased ATP
↓
2
Decreased Na
Pump
Increased Glycolysis
↓
Other Effects
↓
↓
↓
Increased
influx of
calcium,
water, and,
Na, efflux of
K
↓
Decreased
Glycogen
Decreased
pH
Detachment of
ribosomes, etc.
↓
↓
Decreased protein
synthesis
Clumping of
Nuclear
Chormatin
↓
Cellular
swelling, loss
of microvilli,
blebs, ER
swelling
Lipid deposition
D. (pg 9) Understand how reperfusion of ischemic tissue can lead to further tissue damage
The restoration of blood flow to ischemic but otherwise viable tissues results in exacerbated and
accelerated injury. As a result, tissues sustain the loss of cells in addition to those that are irreversibly
damaged at the end of the ischemic episode. This is call ischemia/reperfusion injury that contributes to
tissue damage in myocardial and cerebral infarctions, but is also amendable to therapeutic intervention.
Reperfusion to ischemic tissues may cause further damage by the following means:
1. Restoration of blood flow bathes compromised cells in high concentrations of Ca++ when the cells
themselves cannot regulate themselves. With high intracellular Ca++, this activates proteases (decreased
cytoskeletal and disruption of membrane), endonucleases (nuclear chromatin damage), phospholipases
(decrease phospholipids in membrane), and ATPases (decreased ATP). This all culminates into the loss of
cell integrity.
2. Increase in blood flow will also recruit inflammatory cells which release high levels of oxygen derived
reactive species and damage cell membrane and mitochondrial membrane permeability.
3. Damaged mitochondria yield incomplete oxygen production and thus increase the production of free
radicals. Cells also have compromised antioxidant effects.
E. (pg 9-10) Know what free radicals are, how they are formed, how they injure cells, and examples
of diseases in which free radicals are involved in pathogenesis
Free radical damage underlies chemical and radiation injury, toxicity from oxygen and other gases, cellular
aging, microbial killing by phagocytic cells, inflammatory cell damage, tumor destruction by macrophages,
and other injurious processes. Free radical generation is also a part of respiration and other normal
functions like microbial defense. (Superoxide breaks down water into O2 and hydrogen peroxide)
Free radicals: chemical species with a single unpaired electron in the outer orbital. These states are
unstable and react readily. When made inside cells they attack and degrade nucleic acids and membrane
3
molecules. They initiate autocatalytic reactions: molecules that react with free radicals are converted into
free radicals (chain reaction).
Free radicals generated by: 1. redox reactions: that occur during normal processes like respiration (reduce
02 by adding 4 electrons to generate water) intermediates are formed such as hydrogen peroxide,
superoxide radicals (also produced by intracellular oxidases), and OH∙. Fenton reaction forms free radicals
and is catalyzed by transition metals (Cu and Fe) who donate or accept free electrons. The reduction step of
the Fenton reaction is catalyzed by superoxide ion.
NO can act as a free radical. Absorbing UV light, X-Rays (radiant energy) can hydrolyze water into
hydroxyl (OH∙) and hydrogen (H∙) free radicals. Metabolizing exogenous chemical by enzymes can
produce free radicals.
Injury:
Lipid peroxidation of membranes: double bonds in membrane lipids are vulnerable to attack by O2
deprived free radicals. This reaction yields peroxides which are unable and can lead to chain autocatalysis.
DNA fragmentation: free radicals react with the thymidine in the nuclear/mitochondrial DNA produce
single strand breaks. This DNA damage has been seen in cell killing and malignant transformation of cells
Cross-linking of proteins: Cross-linking promoted by free radicals result in enhanced rates of degradation
or loss of enzymatic activity.
F. (pg 11) Describe and understand 2 examples of how chemical agents injure cells.
Some chemicals act directly by combining with a critical molecular component or cellular component
or cellular organelle. Ex: mercuric chloride poisoning, mercury binds to the sulfhydryl groups of various
cell membrane proteins, causing inhibition of ATPase –dependent transport and increased membrane
permeability. (How antineoplastic agents and antibiotics work by cell damage). The greatest damage is
sustained by the cells that use, absorb, excrete, or concentrate the compounds.
Many other chemicals are not intrinsically biologically active but must be converted to reactive toxic
metabolites, which will then act on target cells. This usually happens by P-450 oxidases in the SER of
the liver and other organs. The metabolites may bind covelantly to proteins and lipids in the membrane
and damage/injuring cells, however free radicals injure the cell the greatest. For instance CCl4 used in the
dry cleaning industry can be converted to free radical CCl3 ∙ in the liver causing phospholipids peroxidation
breaking down the ER. Will see less export of lipids because they cannot make apoprotein (a protein) that
hooks triglycerides to be secreted by liver. These patients get fatty liver from CCl 4 poisening. Eventually
the mitochondria decrease ATP production and the mitochondria start to swell with subsequent calcium
influx and cell death.
G. (pg 14) Define and understand processes of phagocytosis, endocytosis, and pinocytosis
Phagocytosis: Uptake of large matter via engulfment by the cell.
Endocytosis: Uptake of materials from the external environment.
Pinocytosis: Uptake of soluable smaller macromolecules
With the above the endocytosed vacuoles and their contents fuse with a lysosomes creating phaogolysomes
and degrade by heterophagy (pinocytosis, endocytosis, or phagocytosis) or autophagy (intracellular
organelles/cytosol are put into an autophagic vacuole and form from ribosome free parts of the RER. The
autophagic vacuole then fuses with lysosome creating a autophagolysosome---this is to remove damages
organelles or to remodel—seen in cells undergoing atrophy.) Lysosomes have enzymes that degrade
4
proteins, carbohydrates, while some lipid remain undigested. When undigested debris occurs (lipofuscin
pigments, tattoo ink, etc) lysosomes store these in residual bodies
Lysosomal storage disorders caused by deficiencies in enzymes thus, will see an abnormal accumulation of
intermediate metabolites all over the body (especially neurons).
H. (pg 15) Understand the role of lipofuscin and its role in the aging process
Lipofuscin pigment granules represent indigestible material from intracellular lipid peroxidation. Also
known as the “wear and tear” pigment. It is an insoluable brownish-yellow granular intracellular material
that accumulates tissues (heart, brain, and liver) as a function of age or atrophy. It is a complex of lipid and
protein that results from free radical catalyzed perioxidation of lipids of subcellular membranes. It is NOT
injurious to cells but is a marker of past free radical injury. Aka brown atrophy when you can see the
pigment on gross specimen examination.
I. (pg 16-17) Understand the pathophysiology and causes of hepatic steatosis, recognize examples in
lab case studies
Hepatic steatosis: aka fatty change which is an abnormal accumulation of triglycerides within the
parenchymal cells. Indication of reversible injury, but can be found adjacent to necrosed cells. Most often
seen in the liver since it is the major organ involved in fat metabolism but also seen in kidney, heart, and
skeletal muscle. Causes include: toxins, protein malnutrition, diabetes, obesity, and anorexia, but the
MOST COMMON cause is alcoholism.
Excess accumulation of trigyclerides may result in defects at any step from fatty acid entry to the
lipoprotein exit. (last paragraph pg 16 tell about normal fat metabolism pathway) Hepatotoxins (alcohol
etc) alter mitochondrial and SER function; CCl4 and protein malnutrition decrease the synthesis of
apoproteins; anorexia inhibits fatty acid oxidation; and starvation increases fatty acid mobilization from
peripheral stores.
Mild fatty change: no effect on cellular function
Moderate fatty change: transiently impaired cellular function, reversible unless some vital intracellular
process is impaired
Severe fatty change: fatty change may precede cell death but cells can also die without undergoing fatty
change.
J. (pg 20) Define hemosiderin and hemosiderosis and contrast with hemochromatosis
Hemosiderin: a yellow to brown hemoglobin derived granular pigment that accumulates in tissues when
there is an excess of iron. Iron is normally stored within cells in association with the protein apoferritin
forming micelles. Small amounts are normal in mononuclear phagocytes of bone marrow, liver, and spleen
where excessive RBC breakdown occurs. However, usually if present they are pathologic. Excess of local
iron and hemosiderin results from hemorrhage like with bruising. After lysis of RBCs in area of bruise, the
RBS are phagocytosed by macrophages, hemoglobin is catabolized by lysosomes and the heme iron
accumulates in hemosiderin.
Hemosiderosis: Deposition of hemosiderin in orgasm in tissues due to overload of iron. Found at first in
mononuclear phagocytes of liver, marrow, spleen, and lymph nodes. These deposits then progressively
accumulate the liver, pancreas, heart, and endocrine organs and then the organs become “bronzed” with this
pigment. This occurs when 1) increased absorption of iron 2) impaired ultilization of iron 3) hemolytic
anemias 4) transfusions. Most of the time the accumulation does not damage the parenchymal cells or
impair organ function. However excessive accumulation results in hemochromatosis.
Hemochromatosis: Excessive accumulations of iron (hemosiderin) that results in tissue injury including
liver fibrosis, heart failure, and diabetes.
K. (pg 21-22) Define, understand the pathologic process, and recognize diseases associated with
metastatic versus dystrophic calcification
5
Dystrophic calcification: deposition of calcium salts (along with small amounts of iron, magnesium, and
other minerals) in dead or dying tissues. It occurs in the absence of calcium metabolic derangements (ie:
with NORMAL calcium levels). Comes with any necrotic areas (in atheromas seen in advance
atherosclerosis where the initima is damaged in large arteries and see accumulation of lipids). May be seen
with organ dysfunction or cell injury. It is a cause of aortic stenosis in older people when heart valves are
damage causing cuspal calcification.
Morphology: calcium salts occur as fine white granules or clumps and feel gritty. On histo slides, will see
intracellular and/or extracellular basophilic deposits. In time, heterotrophic bone may develop in those
areas.
Pathogenesis: Iniation (nucleation) and propagation (intra or extracellular processes) start with end product
forming calcium phosphate. Initiation in extracellular sites occurs in membrane bound vesicles (in bone
they occur in matrix vesicle and in pathologic calcification they derive from degenerating cells where the
calcium is concentrated because calcium has affinity for membrane phospholipids and phosphatases
accumulate as a result of membrane bound phosphatases. Initiation in intracellular calcification occurs in
the mitochondria of dead/dying cells that cannot regulate intracellular calcium anymore. Propagation then
occurs and is dependent on the concentration of calcium and phosphate in the extracellular spaces, the
presence of mineral inhibitors, and degree of collagenization. Collagen enhances the rate of crystal growth
but osteopontin (protein that binds calcium) is also involved.
Metastatic calcification: deposition of calcium salts (along with small amounts of iron, magnesium, and
other minerals) in normal tissue. It almost ALWAYS reflects some derangement in calcium metabolism
(ie: hypercalcemia). Thus, high calcium levels exacerbate metastatic calcification.
4 causes of hypercalcemia: 1) increased PTH due to parathyroid tumors or other malignant tumors 2)
destruction of bone due to accelerated bone turnover (Paget’s dz), immobilization, or tumors (increased
bone breakdown associated with leukemias, bone cancer, or multiple myeloma) 3) vitamin D disorders like
Vit D intoxication and sarcoidosis where macrophages activate vitamin D precursor 4) renal failure which
you retain phosphate which leads to secondary hyperparathyroidism
Morphology: Mainly occurs interstitial tissues of the vasculature, kidneys, lungs, and gastric mucosa.
Granules look like dystrophic granules, but do NOT generally cause clinical dysfunction. However,
extensive calcifications in lungs and kidneys may lead to damage.
L. (pg 22-24 Fig 1.19) Provide examples and understand the processes of reversible versus
irreversible cell injury
Within limits, the cell can compensate for disturbance to ATP production, cell membrane, protein synthesis
and DNA and can return the cell to normalcy. However, after repeated and persistent injury causes cells to
surpass the threshold and lead to irreversible cell injury. Irreversible injury causes problems with oxidative
phosphorylation and eventually deplete the ATP supply. Membrane damage is a critical step in the
development of lethal injury and increased intracellular calcium can turn on enzymes which can chew up
proteins, cell membrane components, and DNA.
Irreversible cell injury hallmarks: 1) cannot reverse mitochondrial dysfunction after original injury is
resolved (ie: restoration of blood flow) 2) profound membrane function problems (key of irreversible cell
injury) due to loss of phospholipids, cytoskeletal abnormalities, toxic oxygen radicals, and lipid breakdown.
Usually functional changes occur before morphological changes. Reversible injury ultrastructural
changes: plasma membrane problems (blebbing, distortion of microvilli, loosening of intracellular
attachments), mitochondrial changes (swelling and dense opacities of mitochondria), dilation of the ER
(with ribosome detachment), and nuclear alterations. Reversible injury starts with cell swelling (hydropic
changes) and will see fatty change (from hypoxic/toxic/metabolite injury). Irreversible cell injury:
(necrosis) morphologic changes that follow cell death in living tissue. Get mitochondrial and cell
6
swelling. DNA degeneration with pyknosis, karryohexis, karyolysis. Dead cells increase in eosinophilia
(pink dye) because of increase eosin binding.
M. (pg 25) Understand definitions of karyolysis, karyorrhexis, pyknosis
Pyknosis: nuclear shrinkage and increased basophilia (blue staining)
Karyorrhexis: pyknotic nuclease fragments
Karylysis: basophilia of the chromatin may fade secondary to DNAse activity
N. (pg 25) Compare and contrast liquefactive, caseation, coagulative, fat, and gangrenous necrosis.
Understand what is happening at the cellular level in each of these processes, which organs tend to
exhibit each of them, and recognize examples of diseases in which they play an important role
Liquefactive Necrosis: enzymatic digestion is the primary pattern, due to bacteria or fungal infections.
WBCs accumulate. Hypoxic death of cells within the CNS also exhibit this type of necrosis. This necrosis
completely digests dead cells.
Caseous Necrosis: most often due to TB, it is a cheesy white gross appearance of the central necrotic area.
Granular debris is enclosed in a ring of inflammation. Cannot tell what type of tissue you are looking at
(unlike coagulative).
Coagulative Necrosis: denaturation is the primary pattern recognized, basic outer structure of the cells is
preserved (ie: can tell what cell is dead) for days. Classic characteristic of hypoxic death of cells in all
tissues EXCEPT the brain. See increasing acidosis and the injury causes denaturation of structural and
enzyme proteins blocking cellular proteolysis. Ex: myocardial infarction.
Fat Necrosis: focal areas of fat destruction due to pancreatic injury secondary to the release of pancreatic
enzymes. Causes: pancreatitis, pancreas trauma. Fat is liquefied by pancreatic enzymes. This releases
fatty acids which combine with calcium to produce chalky white aresas (fat supponification). On histo
slides, only shadowy outlines of necrotic fat cells may be seen with calcium deposits and surrounding
inflammatory reaction occurring.
Gangrenous Necrosis: refers to ischemic coagulative necrosis (usually of a limb). When there is a
infection with it is called wet gangrene.
Most of the time, necrotic cells are phagocytized. If not, these cells attract calcium salt and other minerals
leading to dystrophic calcification.
O. (pg 26) Understand the mechanisms and various causes of apoptosis at the subcellular level (Fig.
1.26)
Apoptosis: programmed cell death, cell suicide
Examples of apoptosis at work: destruction of cells in embryogenesis, hormone dependent involution (ie
endometrial thickening during menstruation, pathologic atrophy in prostate after castration, decrease in
breast size/lactation with weaning, rapid turnover cells (intestinal epithelium), deletion of autoreactive Tcells in thymus) If failed to undergo apoptosis, would have tumors or autoimmune dz. Usually involves
single or clusters of cells that have eosinophillic cytoplasm. CELLS SHRINK and form buds with cytosol
or organelles called apoptotic bodies. NO INFLAMMATORY RESPONSE OCCURS.
APOPTOSIS:
Stimulus: physiologic and pathologic factors
Histo appearance: single cells, chromatin condensation, apoptotic bodies
DNA breakdown: internucleosomal
Mechanisms: gene activation, endonucleases, proteases
Tissue reaction: NO inflammation, phagocytosis of apoptotic bodies
Mechanisms of apoptosis:
7
Signaling: due to lack of growth factor, intrinsic programmed event, release of granzymes from cytotoxic
T cells, radiated cells, etc. TNF receptor shares the death domain that when oligimerized leads to
activation of caspases and cascade leading to cell death
Control and Integration: proteins that connect the original death signals to the final execution program
(not sure how indepth he is wanting us to go thru---review pg 27)
P. (pg 28-30) Understand and provide the examples of events that occur in cell aging
As we age, oxidative phosphorylation, synthesis of proteins, nutritional uptake, and the ability to repair
chromosomal damage is reduced. Morphological changes include: irregular nuclei, vacuolated
mitochondria, diminished ER, and distorted Golgi, increased accumulation of lipofuscin, abnormally folded
proteins and AGES (advanced glycosylated end products occur. Involves intrinsic clock of aging, and
extrinsic wear and tear. Intrinsic clock: fibroblast stop dividing after 50 doublings in adults (65 in neonates
and 35 in patients with progeria who age prematurely)
Incomplete replication of chromosome ends (telomere shortening) and clock genes are apart of the
explanations behind intrinsic clock.
Cellular aging mechanisms involve both programmed cell events and the consequences of progressive
environmental injury. Programmed aging assumes a predetermined sequence of events including
repression and derepression of specific genetic programs leading ultimately to senescence.
Ch 2
a. The four cardinal signs of inflammation: calor, rubor, dolor, tumor—understand
their meaning and an explanation for how they occur.
Inflammation- a protective response to eliminate the initial cause of cell injury as well as
the necrotic cells & tissues resulting from the original insult; whole body’s response to
correct/attempt to limit response to a local area. Helps clear infection, and along with
repair, makes wound healing possible. Can cause damage.
Acute inflammation has the four signs. Vascular changes & cellular movement and
activation are the two main components.
1. calor- “heat” in inflamed area; due to increased blood vessel dilation & blood
flow
2. rubor- “redness”; due to increased blood flow in area & vascular dilation
3. tumor- “edema”; 1st blood fluid (plasma) leaks into tissues (called
transudate—i.e., blisters); blood proteins follow secondly, forming exudates
4. dolor-“pain” ; Neutrophils that have migrated to the tissue engulf bacteria
and release mediators that cause pain
b. Understand the sequence of & definitions of terms such as diapedesis, adhesion,
margination (transmigration)., rolling, etc, that occur in cellular & vascular events
of acute inflammation (these will be in sequential order)
(1) Margination & Rolling
Margination- as blood flows into post-capillary venules, it undergoes laminar
flow where RBC’s flow fastest in the central part of the vessel while
8
leukocytes are pushed to the edge of the vessel where they can better interact
with endothelial cells.
Rolling-endothelial cells & leukocytes express selectin molecules which bind
to carbohydrate ligands (Sialyl Lewis-x on leukocytes); E & P Selectin on
endothelial cells, L-Selectin is on leukocytes; Relatively loose adhesions;
molecules are always present but upregulated by mediators upon infection
(2) Adhesion & DiapedesisAdhesion is mediated by molecules of the Ig superfamily receptors (ICAM1, VCAM-1) on the endothelial cells that interact with integrins on
leukocytes.
Diapedesis- movement between endothelial cells; mostly in the venules in
systemic vasculature; leukocytes move from blood into tissues by moving
in between endothelial cells; PECAM-1 is main molecule used. Different
leukocytes use different adhesion molecules
c. Understand the processes of chemotaxis, leukocyte activation, & phagocytosis
(1) Chemotaxis & ActivationChemotaxis- leukocytes move towards infection site due to a chemical
gradient (stronger as it gets closer to infection); chemoattractants include
chemkines, complement, leukotrienes, & bacterial chemicals
Activation- induced by chemotactic factors; includes degranulation of
leukocytes, secretion of lysosomal enzymes, production of AA
(Arachnidionic Acid) metabolites, and modulation of leukocyte adhesion
molecules
(2)Phagocytosis & DegranulationPhagocytosis- 3 steps
1. Recognition & attachment of Ag to WBC
i.) caused by opsonins (Ig, C3b, collectins) on Ag
2. Engulfment & phagosome formation
i.) triggered by leukocyte binding to opsonins on Ag
ii.) psuedopods wrap around particle, form phagosome,
then phagolysosome & leukocyte degranulates
3. Killing & degradation of ingested particle
i.) oxidative burst generates free radicals from NADPH
Oxidase enzyme complex & myeloperoxidase in lysosomes
(from azurephlilic granules); kill & degrade particle;
Granules also contain lysozme, major basic protein, etc to
degrade particle
d. Understand the function, source, local & systemic Effect of Chemical Mediators
of acute inflammation including: Histamine, serotonin, complement, bradykinin,
kallikrein, Hageman factor, interleukins, tumor necrosis factor, platelet activating
factor, chemokines, nitric oxide
1.) histamine- Causes arteriolar dilation & is principal mediator of acute
imflammation; Also causes gaps between endothelial cells & their contraction
(vascular permeability); from mast cells; local mediator
2.) serotonin- sames functions & effects as above; released when platelets
aggregate; local mediator
9
3.) Complement- systemic mediator; from plasma in active & inactive form;
generate MAC to lyse cells, and form opsonins for phagocytosis, serve as
molecules for chemotaxis and increase vascular permeability
4.) bradykinin- systemic mediator in plasma; causes increased vascular
permeability, arteriolar dilation, & bronchial smooth muscle contraction. Causes
pain (dolor!)
5.) kallikrein- systemic mediator in plasma; intermediate in kinin cascade that has
chemotactic activity & activates & amplifies the effects of
6.) Hageman factor- systemic mediator in plasma; key intermediate in the junction
between the kinin cascade & clotting cascade (leads to more bradykinin,
thrombin, & complement); initiates four systems in the inflammatory response 1vasoactive kinins, 2-clotting cascade (thrombin, fibrinolytic peptides, factor x)
inflammatory; 3-fibrinolytic system-degrade clots 4) complement system
6.)interlekin 1 & Tissue necrosis factor (TNF)- produced by activated
macrophages; increase expression of adhesion molecules on endothelial cells,
secrete more cytokines and growth factors, produce eicosanoids and NO,
increased thrombogenicity; Activate fibroblasts which increase the ECM.
TNF—aggregates & actives neutrophils; can lead to tissue damage
Hypotensive effects of septic shock (weak heart beat & smooth muscle
relaxation
Systemic effects of TNF & IL-1: fever, lethargy, release of neutrophils into
circulation, metabolic wasting (cachexia), liver produce more proteins, and
release of ACTH
(7) Platelet activating factor-(PAF) derived from phospholipids of neutrophils,
monocytes, basophils, endothelium, & platelets among others by action of
phosopholipase A2. (induces most aspects of inflammation)
Aggregates platelets and causes degranulation. Causes vasoconstriction &
broncoconstriction, vasodilation and vasopermeability, increases adhesion,
chemotaxis, oxidative burst, & makes other mediators esp eicosanoids.
(8) chemokines- activate & attract leukocytes; form gradient, stimulate
hematopoiesis; are produced by activated macrophages, endothelial cells, &
fibroblasts
(9)nitric oxide- derived from endothelium in blood vessels (also in other places
like CNS, but keep not pertinent here); Causes vasodilation (smooth muscle
relaxes); affects only cells very close by; also reduces leukocyte recruitment to
inflammatory sites, is microbicidal, and increases platelet adhesion, aggregation
& degranulation.
e) Understand the arachidonic cascade (lipoxygenase & cyclooxygenase pathways)
and rationale for how steroids and aspirin work and the vascular & cellular effects
of thromboxanes, prostacyclins, lipoxins, leukotrienes, in acute inflammation.
Arachidonic cascade- Lipoxygenase cascade (leukotrienes & lipoxins)
LTCs-vasoconstriction, vasopermeability, brochoconstiction
Lipoxin-vasodilation, neutrophil inhibitor, stimulate adhesion
10
In neutrophil : enzyme 5-lipoxygenase metabolizes AA in into intermediate. Inter
mediate converted leukotriene (LTA-4). LTA-4 can then form LTB4, a chemoattractant
5-LO + AALTA4LTB4;
In platelet LTA4 converts to lipoxins (pro-and anti inflammatory molecules) or LTC4
(vasoconstriction & vascular permeability)
LTC4LTA4lipoxin
Cyclooxygenase Pathway (prostglandins & thromboxanes)
AAPGG2PGH2  a. other PGs, or b) thromboxanes
PGs—vasodilation, inhibit platelet aggregation
Thrombaxane—vasoconstriction, promotes platelet aggregation
Steroids stop the pathways from the beginning because they prevent phospholipases from
cleaving phospholipids into AA.
Asprin blocks the AA before the cyclooxygenase pathway (no PG, no pain!)
f. Understand the terms resolution, fibrosis, and organization in the context of
outcomes of acute inflammation
Resolution- short injury period, little tissue damage, replace damaged cells; return to
normal functionally & histologically; remove mediators, stop leukocyte invasion, return
vessels to normal
Fibrosis (Scarring)- substantial damage in tissues that will not reform; Fibrinous
exudates are not absorbed and & are organized by connective tissue growth; this causes
fibrosis; Abcess formation (lots of pus) causes scarring
--Acute inflammation may become chronic inflammation
g. Understand the process of granulomatous inflammation, disease categories
associated with this condition, recognition of lab cases, and caseating vs. noncaseating granulomas;
Granulomatous inflammation—a distinctive pattern of chronic inflammation
characterized by aggregates of activated macrophages that assume an epithelioid
appearance; Does not rid body of infectious agent; “walls it off”
often seen in TB (tuberculosis), leprosy, fungi, gumma in syphilis, or from inert foreign
body (breast implant).
Lab-central necrosis (no nuclei in cells) and surrounding epithelioid macrophages; lots of
purple lymphocytes around the whole thing
*Caseating-cheese-like granuloma; central area of necrosis from hypoxia & free radical
injury (like you see in TB); granular looking central area; some liquefactive necrosis
almost
*Non-caseating- lack of central necrosis in granuloma *I’m not sure—didn’t see in book,
so double check these guys
i. Understand and be able to describe & recognize serous, fibrinous, purulent, &
ulcerating inflammation. Be familiar with examples in case studies
11
1. Serous inflammation- characterized by the outpouring of a watery, protein
poor fluid derived from serum or mesothelial cells lining the peritoneal,
pericardial, & pleural cavities. Ex: skin blister from burns, viral infection
2. Fibrinous inflammation- more severe injuries resulting in more vascular
permeability, which allows large molecules such as fibrinogen through the
vessels. Fibrin strands—can be removed by macrophages and restore structure
(resolution) or rerouting of channels (organization). Basically scar formation.
Serous Inflammation:
It is characterized by effusion (the watery outpouring of a watery, relatively
protein poor fluid), which depending on the site of injury derives either from the
serum or from the secretions of the mesothelial cells lining the peritoneal, pleural,
and pericardial cavities. An example of a serous effusion accumulated either
within or immediately beneath the epidermis of the skin is a skin blister resulting
from a burn or viral infection (see figure 2-24A pg. 58).
Fibrinous Inflammation:
It occurs as a consequence of more severe injuries, with a resultant greater
vascular permeability allowing larger molecules (specifically fibrinogen) to pass
the endothelial barrier. Histologically, the accumulated extravascular fibrin
appears as an eosinophilic meshwork of threads or, sometimes, as an amorphous
coagulum (see figure 2-24B pg. 58). Fibrinous exudates may be degraded by
fibrinolysis, and the accumulated debris may be removed by macrophages,
resulting in resolution or the restoration of the normal tissue structure. Failure to
completely remove the fibrin results in organization or the ingrowth of fibroblasts
and blood vessels, leading ultimately to scarring. For example, organization of
fibrinous pericardial exudates forms dense fibrous scar tissue that bridges or
obliterates the pericardial space and restricts myocardial function.
Purulent (Suppurative) Inflammation:
It is manifested by the presence of large amounts of pus, or purulent exudates,
consisting of neutrophils, necrotic cells, and edema fluid. Certain organisms, like
staphylococci, are more likely to induce this localized suppuration and are
therefore referred to as pyogenic. Abscesses are focal collections of pus that may
be caused by deep seeding of pyogenic organisms into a tissue or by secondary
infections of necrotic foci. Abscesses typically have a central, largely necrotic
region rimmed by a layer of preserved neutrophils (see figure 2-24C pg. 58) with
a surrounding zone of dilated vessels and fibroblastic proliferation indicative of
early repair. In time, the abscess may be come completely walled off and
eventually replaced by connective tissue.
Ulceration:
It refers to a site of inflammation where an epithelial surface (skin, gastric
epithelium, colonic mucosa, bladder epithelium) has become necrotic and eroded,
often with associated subepithelial acute and chronic inflammation. This can
occur as a consequence of toxic or traumatic injury to epithelial surface, (example
12
a peptic ulcer) or may be due to vascular compromise (as in foot ulcers associated
with vasculopathy of diabetes). The peptic ulcer of the stomach or duodenum
(see figure 2-24D pg. 58) illustrates the typical findings. There is usually an early
intense neutrophilic infiltrate with associated vascular dilation. In chronic lesions
where there has been repeated insult, the area surrounding the ulcer develops
fibroblastic proliferation, scarring, and the accumulation of chronic inflammatory
cells..
j. Understand the systemic effects of inflammation such as leukocytosis, fever, and
the function of eosinophils in parasitic disease
The systemic effects of inflammation is also called acute phase reaction. The
most important mediators of this reaction are cytokines IL-1, IL-6 and TNF.
They are released in a cascade in which TNF induces IL-1 production which
induces IL-6 production. TNF and IL-1 both act on the thermoregulatory center
of the hypothalamus to induce fever. IL-6 stimulates the production of plasma
proteins such as fibrinogen which causes erythrocyte agglutination.
Leukocytosis or increase white blood cell count is a common feature of
inflammatory responses. The normal WBC count of 4,000-10,000µL increases to
15,000-20,000 µL with inflammation and can soar to as much as 40,000-100,000
µL. Leukocytosis is induced when IL-1 and TNF cause the release of cells from
the bone marrow and results in a “left shift” or higher than normal number of
immature neutrophils in the blood.
The type of infection can dictate the type of defensive cell made in response.
Bacterial infections induce higher neutrophil (PMN’s) production. Parasitic
infections induce eosinophil production.
CHAPTER 3: Wound Healing and Repair
a) Pg 62. Fig 3-2 The Role of the cell cycle as it relates to which types of cells are
capable of regeneration, ex. of labile cells, stable cells, and permanent cells.
1) LABILE CELLS: continuously dividing (and continuously dying). Regeneration
occurs through stem cells with a relatively unlimited ability to proliferate. When
stem cells divide... one daughter cell retains the ability to divide while the other
differentiates into nonmitotic cells that carry out the function of the tissue. Ex:
hematopoietic cells in the bone marrow, the majority of epithelia: Stratified
Squamous Epith: skin, oral cavity, vagina, cervix; Cuboidal Epith: salivary
glands, pancreas, biliary tract; Columnar Epith. GI, uterus, fallopian tubes;
Transitional Epith: uninary tract. Continuous cycle from one mitosis to the
next.
2) STABLE CELLS: Quiescent-low levels of replicative capacity in the normal
state but are capable of undergoing rapid division in response to injury. Ex:
Parenchyma of most solid tissue including liver, kidney, and pancreas;
Endothelial cells lining blood vessels; fibroblast and smooth muscle tissue
13
(extremely important in response to injury) In Go. Are neither cycling nor
dying and can be introduced to re-enter the cell cycle by approp. stimulus.
3) PERMANENT CELLS: terminally differentiated and nonproliferative in
postnatal life. Ex: Neurons and cardiac muscle. Injury to brain and heart are
irreversible and results only in scar tissue. Nondividing permanent cells that
have exited the cycle and are destined to die without further division.
b) Pg 68. (Fig 3-9) Definitions and roles of collagen, elastin, proteoglycans,
fibronectin, and integrins in tissue repair.
1) COLLAGENS are fibrous structural proteins conferring tensile strength.
These proteins are composed of 3 peptide chains braided into a ropelike triple helix.
The fibrillar collagens form a major proportion of the connective tissue in
healing wounds and particularly in scars. Require glycine in every 3rd position to
tightly intertwine.
2) ELASTINS the ability of tissue to recoil and return to a baseline structure
after physiological stress. Especially important in large blood vessels (which must
accom. recurrent pulsatile flow) as well as uterus, skin, and ligaments. Elastic fibers
consist of central core of elastin protein surrounded by a meshlike network of fibrillin
glycoprotein. Require glycine in every 3rd position but have fewer crosslinks than
collagen.
3) PROTEOGLYCANS form highly hydrated compressible gels conferring
resilience and lubrication (cartilage of joints). They consist of large
polysaccharides called glycosaminoglycans linked to a protein backbone (bristles on a
test tube brush). Heparan sulfate (GAG) binds basic fibroblast growth factor (bFGF)
secreted into the ECM. Any subsequent injury to the ECM can release the bFGF,
which can then stimulate recruitment of inflammatory cells, fibroblast activation, and
new blood vessel formation. Proteoglycans serve as reservoirs for growth factors
secreted into the ECM. Syndecan is a cell surface proteoglycan with a
transmembrane core protein and attached extracellular GAG side chains that can bind
bFGF from the ECM and mediate improved interactions with cell surface bFGF
receptors. The cytoplasmic tail of syndecan attaches to the intracellular actin
cytoskeleton and helps to maintain the morphology of the epithelial sheets.
.
4) FIBRONECTIN (an adhesive glycoprotein) is a large disulfide linked
heterodimer synthesized by fibroblast, monocytes, and endothelium and assoc. cell
surfaces, BMs, and pericellular matrix. Can bind to a variety of ECM components
such as collagen, fibrin, heparin, and proteoglycans. It can also attach to cell integrins
via RGD (Tripeptide arginine-glycine-aspartic acid). RGD recognition plays a role
in ECM adhesion.
5) INTEGRINS are a family of transmembrane heterodimeric glycoproteins whose
intracellular domains assoc. with cytoskeletal elements. Bind ECM and interact with
the cytoskeleton at focal adhesion complexes (protein aggregates that include
vinculin, alpha actinin, and talin). This can initiate the production of intracellular
second messenger or can directly mediate nuclear signals. Cell surface receptors for
growth factors also initiate second signals. These are all integrated by the cell to
14
yield various responses, including changes in cell growth, locomotion, and
differentiation.
Pg 70-71. Describe and Understand the process of angiogenesis, neovascularization,
granulation tissue in would healing.
1) ANGIOGENESIS (neovascularization) is when preexisting vessels send
out capillary sprouts to produce new vessels. Critical in healing site of injury,
development of collateral circulations at sites of ischemia, and allowing tumors to
increase is size beyond the constaints of their original blood supply. (Fig 3-11)
4 Steps of Angiogenesis:
i) Proteolytic degradation of parent vessels basement membrane allowing the
formation of a capillary sprout
ii) Migration of endothelial cells from the original capillary toward an angiogenic
stimulus.
iii) Proliferation of the endothelial cells (mitosis) behind the leading edge of
migrating cells.
iv) Maturation of endothelial cells with inhibition of growth and organization into
capillary tubes; this includes recruitment of pericytes (for capillaries) and smooth
muscle cells (for larger vessels) to support the endothelial tube and provide
accessory functions.
-there is an increased permeabliltiy due to intercellular gaps and increased transcytosis.
The increased permeability allows deposition of the plasma proteins (fibrinogen) in
the ECM and provides stroma for fibroblast and endothelial cell ingrowth; this
leads to edema in granulation tissue and explains why edema persist long after the
acute inflammatory response is resolved.
-Factors that induce Angiogenesis: Basic Fibroblast Growth Factor (bFGF) and
Vascular Endothelial Growth Factor (VEGF). bFGF binds to proteoglycans in the BM to
be released when structures are damaged. Receptors for Angiogenic Factors are all with
intrinsic kinase activity and are restricted to endothelial cells. They cause proliferation
and induce endothelial cells to secrete proteinases to degrade the basement membrane,
promote endothelial cell migration and direct vascular tube formation from the expanding
cell population.
-Structural ECM proteins regulate vessel sprouting with interactions with integrins on
migrating endothelial cells
-Nonstructural ECM proteins destabilize cell-ECM interactions to facilitate continued
migration or degrade ECM to allow remodeling.
- Granulation tissue: exhibits numerous blood vessels, edema, and a loose ECM
containing occasional inflammatory cells
d.) Pg. 72- Understand the process and role of fibroblasts in scar formation and
remodeling.
15
Surrounding blood vessels is loose connective tissue made up of fibroblasts,
which are spindle-shaped cells used in wound healing and repair that are targets for
cytokines released by macrophages and play a role in the formation of granulation tissue
and healing. The process of fibrosis occurs in two steps: 1) emigration and proliferation
of fibroblasts into the site of injury, and 2) deposition of ECM by these cells. This
process is driven by many growth factors such as platelet-derived growth factor(PDGF)
and TGF-B (see section “e”). This response is elaborated by inflammatory cells such as
macrophages, which clear extracellular debris, in addition to augmenting a host of
mediators that induce fibroblast proliferation and ECM production. The number of
proliferating fibroblasts and new vessels decreases as they assume the role of increasing
ECM deposition. Important to this process is collagen synthesis by fibroblasts, which
begins early in the wound healing process. As in the process of fibrosis, collagen
synthesis is induced by several growth factors such as PDGF and TGF-B. Net collagen
accumulation depends not only on increased synthesis but also on diminished collagen
degradation. The scar that is formed is composed of inactive fibroblasts, dense collagen,
fragments of elastic tissue and other ECM components.
The degradation of collagens and other ECM components is accomplished by
metalloproteinases which are produced by fibroblasts, macrophages, and neutrophils.
Due to their potential to cause harm on tissues, their activity is closely regulated. They
are usually inactive precursors stored in zymogen granules that must first be activated.
These metalloproteinases in addition to collagenases and their inhibitors are spatially
and temporally regulated in healing wounds and are essential in the repair of injured
sites and in the remodeling of the ECM.
e.) p. 72 – Understand the general concept of major growth factors in wound healing,
specifically EGF and TGF-B.
EGF is mitogenic for a variety of epithelial cells and fibroblasts. EGF stimulates
cell division by binding to a tyrosine kinase receptor on the cell membrane, followed by
phosphorylation and other activation events. TGF-A is homologous to EGF, binds to
EGF receptor, and exhibits biologic activities.
TGF-B is produced in an inactive form by a variety of cell types, including
platelets, endothelium, T cells, and activated macrophages. It must be proteolytically
cleaved by plasmin to become functional. TGF-B is a growth inhibitor for most epithelial
cells. It has varying effects on the proliferation of mesenchymal cells. In low
concentrations it induces the synthesis and secretion of PDGF and so is indirectly
mitogenic. At high concentrations it is growth inhibitory because it blocks the expression
of PDGF receptors. Also stimulates fibroblast chemotaxis and the production of
collagen and fibronectin by cells while inhibiting degradation of the extracellular matrix
by metalloproteinases.
f.) P 74-76 Compare and contrast wound healing by first and second intentions and
understand figure 3-14.
16
First Intention: One of the simplest examples of wound repair is the healing of a
clean, uninfected surgical incision and is referred to healing by first intention. The
incision causes only focal disruption of the epithelial basement membrane and the death
of only a few epithelial and CT cells. Epithelial regeneration predominates over fibrosis
and the narrow space rapidly fills with fibrin-clotted blood; dehydration at the surface
produces a scab to cover and protect the repair site. Within 24 hours neutrophils are
seen at the incision margin and migrate toward the fibrin clot. In 3-7 days these
neutrophils are replaced by macrophages and granulation tissue invades the space. In
the weeks to come there is continued collagen accumulation and increasing collagen
deposition within the scar that is devoid of inflammatory cells and covered by normal
epidermis.
Second Intention: When cell or tissue loss is more extensive the reparative
process is more complex and regeneration of parenchymal cells alone cannot restore the
original architecture. Extensive growth of granulation tissue results and this form of
healing is known as healing by second intention. This form of healing differs from
primary healing in three ways:
1. Large tissue defects intrinsically have a greater volume of necrotic debris,
exudates, and fibrin that must be removed. This leads to a much more intense
inflammatory response.
2. Much larger amounts of granulation tissue are formed.
3. Secondary healing exhibits a phenomenon of wound contraction, a process
due mainly to myofibroblasts.
g) Pg. 76. Define Keloid. Keloid: the accumulation of exuberant amounts of collagen
can give rise to prominent, raised scars which are called keloids. There appears to be a
heritable predisposition of keloid formation and the condition is more common among
African Americans
Chapter 4 (cross reference to Dr. Anderson’s lecture outline on
hemodynamics)
a.
P. 80-82 Definition of edema and how it occurs. Understand how conditions
in Table 4-1 and Fig. 4-2 produce edema.
Edema
1. Increased fluid in the interstitial tissue spaces
2. Figure 4-1 Capillary hydrostatic and osmotic forces are normally balanced so that
there is no net loss or gain of fluid across the capillary bed. Increased hydrostatic
pressure of diminished plasma osmotic pressure leads to a net accumulation of
extravascular fluid. As the interstitial fluid pressure increases, the tissue
lymphatics remove much of the volume eventually returning it to circulation via
the thoracic duct. The ability of the lymphatics to drain the tissue determines if
edema persists
3. Table 4-1 Pathophysiological categories of edema
17
Increased hydrostatic Pressure: localized increase in intravascular pressure may
result from impaired venous return.
Examples
-Congestive heart failure
-Constrictive pericarditis
-Ascites (liver cirrhosis)
-Venous obstruction or compression
-Thrombosis
-External Pressure (eg. mass)
-Lower extremity inactivity with prolonged dependency
Reduced Plasma Osmotic Pressure (Hypoproteinemia): can result from excessive
loss or reduced synthesis of albumin, the serum protein most responsible for
maintaining colloid osmotic pressure.
Examples
-Protein-losing glomerulopathies (nephritic syndrome)
-Liver cirrhosis (ascites)
-Malnutrition
-Protein-losing gastroenteropathy
Lymphatic Obstruction: impaired lymphatic drainage and consequent lymphedema
Examples
-Inflammatory
-Neoplastic
-Postsurgical
-Postirradiation
Sodium Retention: increased salt, with the obligate accompanying water, causes
both increased hydrostatic pressure and diminished vascular colloid osmotic pressure.
Examples
-Excessive salt intake with renal insufficiency
-Increased tubular reabsorption of sodium
-Renal hypoperfusion
-Increased rennin-angiotensin-aldosterone secretion
Inflammation:
Examples
-Acute inflammation
-Chronic inflammation
-Angiogenesis
4. Figure 4-2 Congestive heart failure if associated with reduced cardiac output
and reduced renal perfusion. Renal hypoperfusion triggers the renin-angiotensinaldosterone axis inducing sodium and water retention by kidneys. This is
intended to increase intravascular volume and improve cardiac output with
restoration of renal perfusion. If the failing heart cannot increase cardiac output,
the extra fluid load results in increased venous pressure and edema. Unless
cardiac output is restored or renal water retention is reduced a cycle of renal fluid
retention and worsening edema ensures. Treatments include salt restriction,
diuretics, or aldosterone antagonists
BE ABLE TO RECOGNIZE EDEMA IN LAB CASES!
18
b. P. 82-83 – Describe the process of chronic passive congestion and be able to
recognize examples in lab cases.
Chronic passive congestion, or long standing congestion, is the stasis of poorly
oxygenated blood that may cause chronic hypoxia, which can result in
parenchymal cell degeneration or death, sometimes with microscopic scarring.
* Capillary rupture at these sites of chronic congestion may also cause small foci
of hemorrhage; breakdown and phagocyhtosis of the red cell debris can
eventually result in small clusters of hemosiderin-laden macrophages.
***Note! There are tons of pics online in Lab 4 to look at and be able to
recognize that they are chronic passive congeston (in the liver especially!)
c. P. 82-83. Definitions
1. Hyperemia-indicates a local increased volume of blood in a particular tissue. It
is an ACTIVE process (differentiates it from congestion, which is passive),
resulting from augmented blood flow due to arteriolar dilation: ex-sites of
inflammation, or in skeletal muscle during exercise. It causes red skin because of
engorgement with oxygenated blood.
2.
Transudate-protein poor edema fluid that occurs in hydrodynamic derangement
(?). Specific gravity is 1.012. It is basically just serum in there, ex: a
“strawberry” or carpet burn. Contains no WBC’s.
3. Exudate-protein rich inflammatory edema that occurs due to increased vascular
permeability to site of inflammation. Specific gravity more than 1.020. Contains
WBC’s . Ex: the look of your finger after a bad splinter
4. Hemorrhage-indicates extravasation of blood due to rupture of blood vessels.
Capillary bleeding can occur with chronic congestion. An increased tendency to
hemorrhage from usually insignificant injury is seen in clinical disorders called
hemorrhagic diatheses. Rupture of large arteries or veins is usually due to
vascular injury (trauma, atherosclerosis, inflammation, or neoplastic erosion of
vessel wall). Hemorrhages can be external or enclosed within a tissue. The
clinical significance of hemorrhage depends on the volume and rate of blood loss.
Losses up to 20% of blood volume rapidly or slow losses of larger amounts have
little impact on normal adults; greater losses could result in hemorrhagic
(hypovolemic) shock. Site of hemorrhage is important: subcutaneous- trivial
maybe, but in the brain stem it could be deadly. Loss of iron and subsequent iron
deficiency anemia, occurs in chronic or recurrent external blood loss (peptic ulcer
or menstrual bleeding). In contrast, when RBC’s are retained, as in hemorrhage
into body cavities or tissues, the iron can be neutralized for hemoglobin synthesis.
19
5. Hematoma-The accumulation from hemorrhage. It can be insignificant (bruise)
or may accumulate sufficient blood to cause death (massive retroperitoneal
hematoma resulting from rupture of a dissecting aortic aneurism.
6. Petechiae- (1-2mm) minute hemorrhages into skin, mucous membranes
(including eye-sclera), or serosal surfaces. Associated with locally increased
intravascular pressure, low platelet counts (thrombocytopenia), defective platelet
function, or clotting factor deficiencies. Patient may have abnormal platelets or
not enough platelets to plug the small holes in vessels.
7. Purpuras- (3-5mm) slightly larger hemorrhages than petechiaes- may be
associated with same disorders as petechiae- as well as with trauma, vascular
inflammation (vasculitis), or increased vascular fragility.
8. Ecchymoses- (1-2cm) larger subcutaneous hematomas (bruises). RBC’s in these
local hemorrhages are degraded and phagocytosed by macrophages. The
hemoglobin (red-blue) is then enzymatically converted into bilirubin (blue-green)
and eventually into hemosiderin (golden-brown). To determine age of bruise:
red=new, purple/black/brown=old.
9. Thrombocytopenia-low platelet counts- can be a factor in causing petechiae and
purpura.
10. Hemarthrosis-large accumulations of blood in your joints- could develop
jaundice from massive breakdown of RBC’s and systemic release of bilirubin.
11. Hemothorax-large accumulations of blood in your thorax - could develop
jaundice from massive breakdown of RBC’s and systemic release of bilirubin.
12. Hemopericardium-large accumulations of blood in the pericardium- could
develop jaundice from massive breakdown of RBC’s and systemic release of
bilirubin.
d. pages 84-90 – Understand the process of normal hemostasis – role of endothelium,
coagulation factors and platelets. It is not necessary to memorize coagulation
cascade. Understand extrinsic versus intrinsic systems in general terms. Describe
the role of fibrogen, fibrin, and antithrombins.
Normal Hemostasis- maintains blood in a fluid, clot free state. Also includes the
process of forming a localized hemostatic plug. Depends on three things:
vascular wall, platelets, and coagulation cascade.
Sequence following injury
a. First there is a brief period of arteriolar vasoconstriction due to the release
of endothelin from the endothelium. Has a transient effect and will need
activation of platelets and the coagulation system or bleeding will resume.
20
b. When platelets contact the extracellular matrix (ECM) they adhere and
become activated. This involves a shape change and release of secretory
granules. This recruits more platelets (aggregation) that form a hemostatic
plug. This is primary hemostasis. Platelets adhering to exposed collagen
activates the intrinsic clotting pathway.
c. Tissue Factor (procoagulant factor synthesized by the endothelium) is
released at the site of injury and along with platelets, activates the
coagulation cascade. More specifically, the extrinsic clotting pathway.
During the coagulation cascade, thrombin cleaves fibrinogen into insoluble
fibrin forming a fibrin meshwork. Thrombin also recruits more platelets
and cause more secretory granules to be released by platelets. This is
called secondary hemostasis.
d. Finally, polymerized fibrin and platelet aggregate form a solid, permanent
plug. At this stage, counter-regulatory mechanisms (tissue plasminogen
activator [tPA]) limit the plug to just the site of injury by breaking down
excess fibrin.
Endothelial Cells – modulate opposing aspects of normal hemostasis. Without
injury, they exhibit anti-platelet, anti-coagulant, and fibrinolytic properties to
maintain the blood in a fluid state. After injury, they have pro-coagulant
functions. Endothelium can be activated by infectious agents, hemodynamic
factors, plasma mediators and by cytokines as well as cell wall injury.
a. Antithrombotic Properties- Intact endothelium prevents platelets from
contacting the ECM and becoming activated. After injury, endothelial cells
synthesize prostacyclin (PGI2) and nitric oxide (NO) which inhibit
platelets from adhering to surrounding non-injured endothelium. Synthesis
of these vasodialators are under the influence of thrombin and cytokines
produced during coagulation. Also endothelial cells use adensosine
diphosphatase to degrade ADP which is needed for platelet aggregation.
Thrombomodulin, a thrombin receptor on the surface of the endothelium,
binds thrombin and activates anticoagulant protein C. This turns off
clotting by cleaving factors Va and VIIIa. Endothelial cells synthesis tPA
to clear away fibrin deposits.
b. Prothrombotic Properties- vonWillenbrand Factor (vWF) is a normal
product of the endothelium found in plasma. After injury, it is an essential
cofactor that facilitates the platelets sticking to the subendothelial collagen
and other surfaces. vWF makes the interaction strong enough to withstand
the shearing force of flowing blood. Also after injury, endothelial cells
secrete tissue factor in the circulation that activates the extrinsic clotting
pathway.
21
Platelets- circulate as membrane bound, smooth discs expressing glycoprotein
receptors for integrins. They contain two types of secretory granules.
1. alpha granules have P-selectin on their membrane. Inside they contain
fibrinogen, factors V and VIII, platelet factor 4, platelet derived growth
factor (PDGF), and transforming growth factor alpha (TGF).
2. delta granules or dense bodies, contain ADP and ATP, ionized Calcium,
histamine, serotonin, and epinephrine.
On contact with ECM, platelets undergo shape change, adhesion, secretion and
aggregation. Secretion occurs soon after adhesion. The release of dense bodies is
important because Calcium is required for the coagulation cascade. ADP is a
potent mediator of platelet aggregation. Platelet activation results in surface
expression of phospholipids complexes that become binding sites for Calcium and
coagulation factors in the intrinsic clotting pathway. Aggregation follows
adhesion and secretion. Platelets secrete ADP (imp. for the formation of a
primary plug) and thromboxane A2 (TXA2). TXA2 is a vasoconstrictor that
stimulates aggregation by slowing down blood flow.
Coagulation Cascade – A series of enzymatic conversions culminating in the
formation of thrombin. Thrombin converts soluble fibrinogen into insoluble
fibrin. Cascade is divided into the intrinsic pathway (stimulated by the exposure
of collagen and the activation of Hageman factor XII) and the extrinsic pathway
(stimulated by the endothelium releasing tissue factor). These two pathways
converge where factor X is activated. Once activated, the coagulation cascade
must be restricted to local site of injury to prevent occlusion of the vessel.
Antithrombin inhibits thrombin. Protein C and S are two Vitamin K dependent
proteins that inactivate thrombin as well. Plasminogen activators (tPA) form
plasmin from plasminogen that breaks down fibrin.
Regardless of whether the coagulation cascade is initiated by an internal
substance to the blood vessel (collagen exposure) or an external substance to the
blood vessel (tissue factor also known as thromboplastin), they both follow the
same common pathway after factor X is activated.
e. P. 90 – Pathologic thrombosis – understand and describe Virchow’s triad.
Virchow’s Triad: Three primary influences that predispose to thrombus
formation.
1. Endothelial Injury: loss of endothelium leads to the exposure of
subendothelial collagen, adherence of platelets, release of tissue factors,
and local reduction of PG2 and PA (basically exposure of collagen leads
to clotting cascade). This dysfunctional endothelium induces a greater
amount of procoagulant factors and a smaller amount of anticoagulant
factors. Some sources of endothelial injury include: breakage of
22
atherosclerotic lesion, hypertension, bacterial endotoxin,
hypercholesteremia, cigarette smoke.
2. Abnormal blood flow: Normal blood flow is laminar such that platelets
are separated from the endothelium by plasma. Turbulent blood flow can
cause endothelial injury and local pockets of stasis that contributes to
arterial and cardiac thrombosis. Turbulence and stasis thus:
a. Disrupt laminar flow and allow platelets to contact endothelium
b. prevent dilution of activated clotting factors by fresh flowing blood
c. retards the inflow of clotting factor inhibitors and permits the build up
of thrombi
d. Promote endothelial cell activation
Examples: aneurysms cause stasis; MI’s have areas of non contractile
myocardium thus adding to stasis;
3. Hypercoagulability: any alteration of the coagulation pathways that
predisposes to thrombosis with primary (genetic) and secondary (acquired)
disorders. Examples: primary: factor V mutations, prothrombin
mutations; secondary: prolonged bed rest or immobilization
f. P. 92-95 – Understand definitions and risk factors for phlebothrombosis,
pulmonary thromboembolism, disseminated intravascular coagulation, embolus,
thrombus, mural thrombus, fat embolism, air embolism, amniotic fluid embolism,
infarction.
Phlebothrombosis
Definition: is venous thrombosis that is almost always occlusive; often creates a
long cast of the vein lumen. These form in slowly moving venous blood and tend
to contain enmeshed erythrocytes and are therefore AKA red, or stasis, thrombi.
Risk factors:
1. Most commonly (90%) of cases affects the veins of the lower extremities
2. cardiac failure
3. trauma, surgery, and burns usually result in decreased physical activity,
injury to vessels, release of procoagulant substances from tissues
Pulmonary thromboembolism
Definition: in 95% of these venous emboli originate from deep leg vein thrombi
above the level of the knee and pass into pulmonary vasculature; right ventricular
failure occurs with 60% or more of the pulmonary circulation is obstructed with
emboli
Risk factors:
1. Deep leg vein thrombi about the level of the knee
2. A patient who has one pulmonary embolus is at high risk of having more
Disseminated intravascular coagulation
Definition: DIC, the sudden or insidious onset of widespread fibrin thrombi in the
microcirculation; NOT a primary dz, but rather an initially thrombotic disorder
23
that can evolve into a serious bleeding disorder that is secondary to the
widespread activation of thrombin
Risk factors:
1. Obstetric complications
2. Advanced malignancy
Embolus
Definition: a detached intravascular solid, liquid, or gaseous mass that is carried
by the blood to a site distant from its point of origin; unless otherwise specified,
assume it to be thrombus in origin.
Risk factors: ischemic necrosis of down stream tissue known an infarction.
Thrombus
Definition: can develop anywhere in the cardiovascular system resulting at a site
of endothelial injury or turbulence; venous thrombi usually originate in an area of
stasis
Risk factors:
1. Primary (Genetic) = Factor V mutations, Antithrombin III deficiency,
Protein C or S deficiency
2. Secondary (acquired) = prolonged bed rest/immobilization, myocardial
infarction, tissue damage, cancer, prosthetic cardiac valves, disseminated
intravascular coagulation, Lupus anticoagulant
Mural thrombus
Definition: arterial thrombi that arise in heart chambers or in the aortic lumen that
are applied to the wall of the underlying structure
Risk factors:
1. Abnormal myocardial contraction (arrhythmias, dilated cardiomyopathy,
or myocardial infarction)
2. Injury to the endomyocardial surface (myocarditis, catheter trauma)
Fat embolism
Definition: the fat is released by marrow or adipose tissue injury and enters the
circulation by the rupture of the marrow vascular sinusoids or rupture of venules;
it is characterized by pulmonary insufficiency, neurologic symptoms, anemia, and
thrombocytopenia, and is fatal in 10% of cases
Risk factors:
1. soft tissue trauma
2. burns
3. fractures of long bones (which have fatty marrows)
Air embolism
Definition: gas bubbles within circulation can obstruct vascular flow and cause
distal ischemic injury; examples include obstetric procedures or as a consequence
of chest wall injury and decompression sickness
24
Risk factors:
1. Gas bubble within the circulation that can obstruct vascular flow (and
cause distal ischemic injury)
2. Air can enter during: Obstetric procedures or as a result of chest wall
injury
3. Sudden changes in atmospheric pressure (decompression sickness)…more
chronic form=caisson disease
4. Gas bubbles in skeletal muscles and supporting tissues in and around
joints (bends)
5. Gas bubbles leading to respiratory distress (chokes)
Amniotic fluid embolism
Definition: infusion of the amniotic fluid into maternal circulation via a tear in the
placental membranes and rupture of uterine veins; this condition is fatal in 80% of
the cases; onset is characterized by sudden severe dyspnea, cyanosis, and
hypotensive shock, followed by seizures and coma.
Risk factors:
1. Labor and immediately postpartum
Infarction
Definition: an area of ischemic necrosis caused by occlusion of either the arterial
supply or the venous drainage in a particular tissue
Risk factors:
1. Thrombotic or embolic events related to arterial occlusion
2. Occasionally, local vasospasm, swelling of an atheroma secondary to
hemorrhage within a plague, or extrinsic compression of a vessel such as
in the case of a tumor
g.
Understand causes and give examples of red vs. white infarcts.
(Answer from the book only-I don’t have these scripts yet!)
(pg 97) An infarct is an area of ischemic necrosis caused by occlusion of either
the arterial supply or the venous drainage in a particular tissue and they can be
classified on the basis of their color (reflecting the amount of hemorrhage) or the
presence or absence of microbial infection.
Red (hemorrhagic) infarcts occur:
1. with venous occlusions (as in ovarian torsion) – infarcts caused by venous
thrombosis are likely in organs with a single outflow channel (testis, ovary)
2. in loose tissues (lung) that allow blood to collect in the infracted area
3. in tissues with dual circulations (lung and small intestines) which permit the
flow of blood from the unobstructed vascular channel into the necrotic area—
(but not enough to rescue the necrotic tissue)
4. in tissues that were previously congested because of a sluggish venous
outflow
25
5. when flow is re-established to a site of previous arterial occlusion and necrosis
(ex: fragmentation of an occlusive embolus or angioplasty of a thrombotic
lesion)
White (anemic) infarcts occur:
1. with arterial occlusions
2. in solid organs (heart, kidney, spleen) where the solidity of the tissue limits the
amount of hemorrhage that can seep into the area of ischemic necrosis from
adjoining capillary beds
h. pg 98-102 Describe the clinical features, pathogenesis, stages, and types of shock.
1. Shock
A. Shock means cardiovascular collapse- final common pathway for a
number of clinically lethal events including, severe hemorrhage, trauma,
burns etc.
B. Shock definition- shock constitutes systemic hypoperfusion (low
perfusion) due to a reduction in either 1) cardiac output or in the 2)
effective circulating blood volume. First comes hypotension, then
impaired tissue perfusion and then cellular hypoxia.
2. 3 Categories of Shock
A. Cardiogenic
1. Results from myocardial damage (think infarction or
arrhythmias
B. Hypovolemic
1. Result from loss of blood (hemorrhage) or plasma (severe burns)
2. Lungs seldom affected b/c they are resistant to hypoxic injury
C. Septic
1. Caused by systemic microbial infection (local infection that has
spread)
2. Most commonly caused by gram neg. infections (endotoxic
shock) but can occur with gram post. Or fungal infections
3. Pathogenesis of Septic Shock
A. 25-50% mortality rate, ranked 1st in cause of death in ICU, cases
increasing in frequency due to improved life support for high-risk patients.
B. Again, septic shock is a local infection that has spread into bloodstream
C. Most common form is endotoxic shock caused by endotoxin-producing
gram neg. bacteria
1. Endotoxin found in LPS (lipopolysaccarides) a toxic fatty acid
found in cell wall.
D. All the effects of septic shock can be reproduced with injection of LPS
alone
E. LPS binds to CD14 on monocytes, macrophages, neutrophils, this
results in an intracellular signaling via the TLR (toll like receptor) which
activates cytokines such as IL-1 and TNF. LPS can also trigger
complement which triggers bacterial eradication.
26
F. In low doses, LPS through TLR triggers the innate immune system to
eradicate microbes. The secondary effects of LPS, however, can cause
severe pathological changes including fatal shock.
G. In higher doses, LPS also cause release of the cytokine-induced
secondary effectors NO (nitric Oxide) and PAF (platelet activating factor).
In addition, systemic effects of TNF are seen like fever
H. High levels of LPS result in:
1. Systemic vasodialation
2. Diminished myocardial contractility
3. Widespread endothelial injury and activation
4. Activation of the coagulation system culminating in DIC
(disseminated intravascular coagulation
I. Toxic Shock Syndrome-similar to septic shock but caused by bacteria
proteins called super antigens.
4. Stages of Shock
A. Shock is a progressive disorder that if uncorrected leads to death
B. The following stages are best documented in hypovolemic shock BUT
are common in the other forms of shock
C. 1st stage-non-progressive stage
1. Reflex mechanisms are active (baroreceptors, rennin-angiotensin
system, ADH release etc) to keep cardiac output up to maintain
vital organ blood perfusion
2. Net effect is tachycardia, peripheral vasoconstriction, and renal
conservation of fluid
D. 2nd stage-progressive stage
1. Characterized by tissue hypoperfusion
2. Onset of circulatory and metabolic imbalance
3. Anaerobic glycolysis and lactic acid buildup
4. Blood pools in microcirculation in periphery worsening cardiac
output
5. Clinically patient may become confused and their urinary output
declines
E. 3rd stage- irreversible stage
1. Sets in after body has cellular injury so severe that even if
hemodynamic effects are fixed survival is not possible
2. Clinically patient has complete renal shutdown
5. Clinical Correlations
A. Antibodies to TNF and IL-1 or pharmacological inhibitors to NO and
PAG have been shown to protect against septic shock
B. Hypovolemic and Cardiogenic Shock symptoms
1. Hypotension, weak rapid pulse, Tachycardia cool clammy skin
C. Septic Shock Symptoms
1. Warm flushed skin due to vasodialation
27
LOOK AT FIGURE 4-20 PG. 101 SHOWS FLOW FROM LOCAL
INFLAMMATION TO SYSTEMIC EFFECTS TO SEPTIC SHOCK
Chapter 5 (cross reference to Dr. Lorenz’ and Dr. Bucy’s
outline on immunopahtology and HIV/AIDS)
5a. Compare and contrast innate immunity vs. adaptive and give examples of each.









Innate Immunity
Hours to days
Preformed
No Memory
Ex. 1: Monocytes/Macrophages
o Phagocytosis
o Ag Presenting Cell
o Secret soluble mediators
Ex. 2: Neutrophils
Ex. 3: Soluble Mediators
o Lectins (CHO Ligands)
o Complement
Ex. 4: NK
o Direct killing of virus/tumor
cells
o 2 Receptor Hypothesis
 KIRS/KARS
o Ab Dependent Cellular
Cytotoxicity (ADCC)
o Cytokine Production (IFN-)
Ex. 5: Innate subset of T Cells
o NK-T cells
 Produce IL-6
o Gamma Delta T Cells
 Located in epithelium
Ex. 6: Innate subset of B Cells
o B-1 Cells
 Located in peritoneal
cavity
 Produce Ab
independent of T cells
28
Adaptive Immunity
 7-10 days
 Specificity
o Generated in response to
foreign Ag
 Memory
 Ex. 1: B Cells (Humoral Immunity)
o Abs / Igs
 Diversity
 Opsinization
 Neutralization
o Clonal Selection
o Mature in Bone Marrow
 Ex. 2: T Cells (Cellular Immunity)
o TCR
 Diversity
o Clonal Selection
o Mature in Thymus
o Recognize Ag in MHC
context
o Types: Helper, Cytotoxic
b. P. 104 Describe the role of T cells in the immune response, CD4+, CD8+
T cells
They direct cellular immunity and are essential for inducing B cells. Make up 6070% of circulating lymphocytes. Major lymphocytes in PALS and lymph node
interfollicular zones.
Each T cell has a specific T cell receptor (TCR) generated by somatic rearrangement
and recognizes a unique processed peptide. One can analyze the TCR gene
rearrangements and use data as a marker for T-lineage which is used to classify
malignancies. Also, the unique DNA rearrangement allow for distinguishing between
polyclonal and monoclonal proliferations. TCR is usually a protein hetero-dimer (
chains) with a variable region (binds target peptide) and a constant region (interacts with
associated signal molecules). There are a few - heterodimers in peripheral blood and
many at the mucosal surfaces. They can recognize non-proteins and may maintain
immunity at the interfaces.
TCRs are noncovalently linked to CD3 and two zeta chains at the TCR constant
regions. These molecules transduce intracellular signals after T cell ligation.
CD4 (on the T helper cell, 60% of T cells) binds invariant portion of MHC II, CD8
(cyto-T cells, 30% of T cells) binds MHC I. They act as coreceptors.
Two signals required for complete activation:
1. TCR with CD4 or CD8 bind appropriate MHC peptide antigen complex
2. delivered by CD28 on T cells with co-stimulatory CD80 or CD86 (B7) on
APCs
Absence of #2 will cause T cell to undergo apoptosis/be unreactive (anergic)
Thelper1 cells – secrete cytokines (such as IL-2 and IFN-) that help direct cellmediated immune responses (macrophage, NK cells)
Thelper2 cells – secrete cytokines (such as IL-4, IL-5, and IL-10) that antagonize TH1
and/or promote humoral immunity (synthesis of IgE)
Tcytotoxic cells are also divided into subclasses.
c. Describe the roll of B cells, plasma cells, antibody function in the immune
response.
B lymphocytes are present in bone marrow as well as lymph nodes, spleen, &
tonsils. Upon stimulation, B cells form plasma cells that secrete
immunoglobulins, which mediate humoral immunity. Each B cell receptor has a
unique antigen specificity.
d. pg 106-107 – Describe the role of macrophages, dendritic cells and NK cells in the
immune response.
Role of Macrophages (found everywhere in tissue)
 Express class II MHC so are central players in processing and presenting antigens
 produce cytokines so they are important effector cells in cell-mediated
immuninty
29

phagocytose microbes coated with antibody or complement this makes them
important for humoral immunity
Dendritic Cells:
cells with fine dendritic cytoplasmic properties. There are two types
1. Interdigitating dendritic (called langerhans in epidermis) that are non phagocytic
that express high levels of MHC class II and costimulatory molecules (found in
lymphoid tissues and interstitium of nonlymphoid organs)
2. Follicular dendritic cells bear Fc receptors for IgG antibodies(found in germinal
centers of lymphoid follicles in spleen and lymphnodes)
NK cells: Contain abundant azurophilic granules and are able to lyse a variety of
tumor cells, virally infected cells, and some normal cells without prior sensitization.
Natural Killer cells recognize reduced number of MHC class I receptors caused by
cell infection. If cells express MHC class I then KIR(killer inhibiting receptor) on
NK cells surface inhibits the cell.
e. p. 107-110 – Understand the concept of HLA and that particular haplotypes can
be associated with various diseases. BE able to recognize examples of diseases
associated with particular HLA haplotypes.
First, here is a brief overview of the MHC and HLA complex.



HLA (human leukocyte antigen) complex is a cluster of genes that code for
histocompatibility molecules that are located on a small segment of chromosome
6.
HLA system is highly polymorphic (several alternative forms of the gene are
available), and this makes them a formidable barrier to organ transplantation.
Based on chemical structure, tissue distribution, and function, MHC gene
products fall into three categories:
o MHC Class I: coded by three closely linked loci designated HLA-A,
HLA-B, and HLA-C. They are heterodimers with an alpha and a beta
chain. They present to CD8+ T cells. Present on all nucleated cells.
o MHC Class II: coded by genes in the HLA-D region where there are
three subregions: DP, DQ, and DR. They are heterodimers that also
contain an alpha and a beta chain. They are expressed on APCs and on B
cells, and they are presented to CD4+ T cells.
o MHC Class III: Includes complement components (C2, C3, and Bf) and
genes for tumor necrosis factor (TNF) and lymphotoxin(TNF-β).
Now, here is the significance of the Histocompatibility Molecules.
30


Any individual inherits one HLA allele from each parent and expresses two
different molecules for every locus. So cells can express six different class I HLA
molecules, three maternal and three paternal.
Similarly, an individual will express maternal and paternal alleles of the MHC II
loci. Each MHC II expressing cell can have over 20 different MHC II molecules.
Different MHC alleles bind to different peptide fragments depending on the
particular amino acid sequence of a give peptide.
These are lists of diseases that are associated with HLA.

Diseases linked to HLA can be grouped in 3 categories:
o Inflammatory diseases. Ex.: ankylosing spondylitis and several
postinfectious arthropathies associated with HLA-B27
o Inherited Errors of Metabolism. Ex.: 21-hydroxylase deficiency (HLABw47)
o Autoimmune Diseases. Ex.: autoimmune endocrinopathies associated
with certain DR alleles.

The mechanisms underlying these associations are not understood. In some cases,
the linkage results from the fact that the relevant disease-associated gene maps
within the HLA complex (e.g. 21-hydroxylase). In the case of immunologically
mediated disorders, it seems likely that the role of MHC II molecules in
regulating the immunoresponsiveness may be relevant.
Shown below is Table 5-1 from page 109 in the book. This gives a list of diseases
that are associated with selected HLA types.

Disease
Ankylosing spondylitis
Postgonococcal arthritis
Acute anterior uveitis
Rheumatoid arthritis
Chronic active hepatitis
Primary Sjögren syndrome
Insulin-dependent diabetes*
21-Hydroxylase deficiency
HLA Allele
B27
B27
B27
DR4
DR3
DR3
DR3
DR4
DR3/DR4
Bw47
Relative Risk
87.4
14.0
14.6
5.8
13.9
9.7
5.0
6.8
14.3
15.0
*Insulin-dependent diabetes goes with three of the alleles (DR3, DR4, and
DR3/DR4).
f. P. 110-111 – Define Cytokines, understand importance of IL-1, II-2, TNF-alpha,
interferon-gamma, and superantigens in the immune response.
31
Cytokines are low molecular, short acting, soluble polypeptides that can be
secreted by lymphocytes, effector cells, APC’s, and monocytes. Cytokines can act
in autocrine, paracrine, and endocrine fashion. They can also be redundant(many
have the same function), pleiotropic ( act on many cells), and antagonistic ( they
can oppose each others actions).
IL -2 (interleukin 2)
-activates T cells by binding to high affinity receptors. Many times the IL2 receptor can be blocked by Ab to treat rejection of a transplanted organ,
thus preventing T-cell activation.
-Can act in an autocrine fashion when produced by T-cells, can feedback
and promote T-cell growth.
- Regulate the growth of T cells, B cells, and NK cells
IL -1 (interleukin 1) and TNF-alpha
- produces the acute phase of inflammation
- can be produced by virtually any cell
- mediate innate immunity
- protect against viral infection
IFN-gamma – activates macrophages and inflammatory cells
Superantigens – are a group of microbial proteins that bind to MHC II molecules
outside the antigen binding region. These proteins can activate a large number of
T cells via the T cell receptors resulting in widespread release of cytokines (i.e.
TNF and IL-2). These superantigens are characterized by staphylococcal
exotoxins that can cause diseases such as: food poisoning, scalded skin syndrome,
and toxic shock syndrome.
g.
(Pages 111-121) 4 types of immune responses--understand processes of each,
recognize and give examples of each, know representative lab cases. Be able
to classify various conditions into which type of immune response they
represent.
Hypersensitivity--defined as abnormal or excessive sensitivity to an antigen.
Category What
Examples
Relevant
Happens?
Lab Cases
Type I

(Allergy &
Anaphylaxis)


rapid tissue response (within
minutes) after interaction between
an allergen with IgE antibody
previously bound on the surface of
mast cells (found in tissues) and
basophils (found in blood) in a
sensitized test.
can be local or systemic (depending
upon route of antigen exposure)
two well-defined phases:
(1) Initial Phase
-evident 5-30 minutes after
exposure; ends within an hour
32
Systemic









Systemic anaphylaxis
uticaria (hives)
skin erythmea
respiratory deficiency
(pulmonary
bronchoconstriction)
hypersecretion of mucus
vomiting
abdominal cramps
diarrhea
systemic vasodilation
(anaphylactic shock)
*There are no lab
cases available
for Type I.
-vasodilation
-vascular leakage
-smooth muscle spasm
(2) Late Phase Reaction
-sets in 2-8 hours later; lasts
several days
-more intense infiltration by
eosinophils and inflammatory
cells
-mucosal epithelial cell damage
Process

mediated by IgE

initial expose to an antigen
(allergen)

allergen stimulates CD4+
(TH2) cells

CD4+ cells secrete cytokines
(IL-4 and IL-5) that, in turn,
case IgE to be made by B cells.
The cytokines also act as mast
cell growth factors and
recruit/activate eosinophils.

IgE antibodies bind high
affinity Fc receptors on mast
cells and basophils. Now these
cells are "armed."

the individual is now primed to
develop Type I
hypersensitivity

reexposure to the antigen
triggers a cascade

several powerful mediators are
released

you get degranulation with
discharge of primary mediators

you also get synthesis and
release of secondary mediators
Primary Mediators

also known as preformed mediators

after IgE is triggered, primary
mediators within the mast cell
granules are released

they initiate the early events in Type
I reactions

histamine released
-increases vascular permeability
-vasodilation
-bronchoconstriction
-increased mucus secretion

other mediators released
-adenosine
-chemotactic factors for
neutrophils and eosinophils
-heparin
-neutral proteases


Local





circulatory collapse
death (within minutes if no
intervention)
skin allergies
food allergies
hay fever
certain forms of asthma
You can be genetically
predisposed to localized Type
I reactions (called "atopy")
Secondary Mediators

Lipid mediators

Mast cell produced cytokines (TNF,
IL-1, IL-4, IL-5, IL-6, and
chemokines)
Category
What Happens?
Examples
33
Relevant
Lab Cases
Type II
Hypersensitivity

(Antibody
Dependent)

mediated by Ab directed against
targer Ag on the surface of cells or
other tissue components
3 different mechanisms-complement mediated reactions,
antibody-dependent cell mediated
cytotoxicity, and antibodymediated cellular dysfunciton
Complemented Mediated Reactions

Ab binds to cell surface Ag

this causes fixation of complement
to cell surface

you get lysis via MAC (membrane
attack complex)

cells coated with Ab and C3b
fragments (now opsonized)
susceptible to phagocytosis

circulating blood cells are most
commonly damaged by this
mechanism
Lab #6
-Graves Disease
Complement Mediated Examples

Transfusion Reactions-recipient Ab directed against
donor's blood group and RBCs
destroyed

Erythroblastosis Fetalis (Rh
Incompatibility)--mother's Ab
(if mom is Rh negative) cross
placenta and cause destruction
of the Rh positive baby's RBCs

Autoimmune Hemolytic
Anemia, Agranulocytosis,
Thrombocytopenia--individual
makes Ab against their own
blood cells

Drug Reactions--Ab directed
against a particular drug that is
absorbed to a cell surface (i.e.,
hemolysis after patient is given
penicillin)

Pemphigus Vulgaris--caused by
Ab against desmosome proteins
that disrupt intracellular
junctions
ADCC Examples

no examples given in textbook
Antibody-Dependent Cell Mediated
Cytotoxicity (ADCC)

killing via cells with receptors for
the Fc portion of IgG

targets coated with Ab are lysed
without phagocytosis or
complement fixation

mediated via PMNs, eosinophils,
macrophages and NK cells

mostly IgG, but occasionally IgE
Ab used
Antibody-Mediated Cellular Dysfunction
Examples

Myasthenia Gravis--Ab against
Ach receptors in motor end
plates of skeletal muscle impair
neuromuscular transmission,
resulting in muscle weakness
Antibody-Mediated Cellular Dysfunction

Ab directed against cell surface
receptors

May impair function without
causing injury or inflammation

or...

Ab can stimulate cell function
34
Graves Disease--Ab against
TSH receptor stimulate thyroid
epithelial cells and result in
hyperthyroidism
-Hashimoto's
Thyroiditis
Category
Type III
Hypersensitivity
(Immune
Complex
Mediated)
What
Happens?
Examples
Relevant
Lab Cases

mediated by deposition of Ag-Ab
complexes, followed by complement
activation and accumulation of
PMNs

applies to endogenous and
exogenous Ag

these immune complexes either
form:
-in the circulation and then deposit
in tissues, or
-form at extravascular where Ag
has been planted

can be localized or systemic
Systemic Immune Complex Disease--3 Phases

Phase 1--formation of Ag-Ab
complex in circulation

Lab 6
-Polyarteritis
Nodosa
Systemic Immune Complex Disease
Example

Acute Serum Sickness-individuals would get this
when injected with horse
serum
Phase 2--deposition of immune
complex in various tissues.
Deposition depends on:
-size of the immune complex
(small or intermediate
complexes more likely to get
overlooked by phagocytic cells
and deposit)
-status of phagocytic cells
-location (deposition more likely
at kidneys, joints, skin, heart,
serosal surfaces and small
blood vessels)

Phase 3--initiation of an
inflammatory response at various
sites throughout the body
-fever
-uticaria
-arthralgia (muscle and joint
aches)
-lymph node enlargement
-proteinuria
Local Immune Complex Disease
Example

Arthus Reaction--a localized
area of tissue necrosis
resulting from immune
complex vasculitis. Reaction
evolves over a few hours,
peaking at 4=10 hours. At
the peak, you will see edema
at the injection site with
severe hemorrage followed
by ulceration.
Local Immune Complex Disease

see Arthus Reaction
35
Category
Type IV
Hypersensitivity
What
Happens?


(CellMediated)
Examples
mediated by specifically sensitized T
cells
divided into two basic types-delayed type hypersensitivity
(initiated by CD4+ T cells) and
direct cell cytotoxicity (mediated by
CD8+ T cells)
Delayed Type Hypersensitivity (DTH)

there must first be an exposure of the
individual to the pathogen (tubercle
bacilli, for example)

CD4+ lymphocytes recognize
peptide Ag in association with MHC
Class II antigens on the surface of
monocytes or dendritic cells

this leads to formation of CD4+ cells
of the TH1 type that remain in the
circulation for years.

on subsequent injections, the
memory cells respond to processed
Ag on APCs and are activated.

secretion of TH1 cytokines follows
and drives the DTH response

The following cytokines are most
relevant--IL-12, IFN-, IL-2, TNF
and lymphotoxin

Granulomas--a special form of DTH
when you have persistent and/or
nondegradable Ag. The CD4+ T cell
infiltrate is replaced by macrophages
over 2 to 3 weeks. You get
formation of epithelioid cells which
then fuse to become giant cells.
Aggregated epithelioid cells become
surrounded by a collar of
lymphocytes and become a
granuloma.
T-Cell Mediated Cytotoxicity

sensitized CD8+ T cells kill Agbearing target cells
36
Relevant
Lab Cases
Lab 6
-Tuberculosis
DTH Examples

Tuberculin Reaction--8 to 10
hours after injection of
tuberculin a local area of
erythema and induration
(hardening of tissue) appears.
This peaks at 24 to 72 hours
(thus the "delay"). There is
accumulation of CD4+
helper cells and some
macrophages. Local
cytokine secretion leads to
increased vascular
permeability causing dermal
edema. It also causes fibrin
deposition that causes the
tissue induration.

Contact Dermatitis--tissue
injury resulting from DTH.
It is evoked by contact with
pentadecylcatechol in a
sensitized
T-Cell Mediated Cytotoxicity

no examples given in
textbook
Definition
Time
Controlled
by
What
happens?
Example
Hyperacute
Transplant
Rejection
Preformed antidonor
Ab present in
circulation of host
before transplant
Rapid (min to hr)
Neutrophils in
vessels and glomeruli
Ig and complement
deposits in vessel
wall. Histology: see
widespread acute
arteritis and
arteriolitis, vessel
thrombosis, and
ischemic necrosis
Patient who has had a
lot of blood
transfusions,
multiparous women,
and someone who has
already rejected a
transplant
Acute Transplant
Rejection:
Ab mediated
Antidonor Ab
present
Acute Transplant
Rejection: CellMediated
Mediated by T cells
Days to months
Immunosupressive
Rx
Necrototizing
vasculitis with
endothelial cell
nerosis;
neutrophilic
infiltration;
deposition of Ab,
complement and
fibrin; thrombosis
Days to months
Immunosupressive Rx
Extensive necrosis
of renal
parenchyma
Extensive interstitial
CD4+ and CD8+ T-cell
infiltration with edema
and mild interstitial
hemorrhage. Glomerular
and peritubular capillaries
contain large numbers of
mononuclear cells, which
may also invade the
tubules and cause focal
tubular necrosis. CD8+
cells may also injure the
endothelium, causing an
endothelilitis
Accompanied by clinical
signs of renal failure
Chronic Transplant
Rejection
Acute rejection
occurring multiple
times leading to
destruction of tissue
Months to years
Have mononuclear
cell infiltrates with
plasma cells and
eosinophils.
Vascular changes,
interstitial fibrosis,
and loss of renal
parenchyma
Kidney can no longer
tell kidney b/c
fibrosis has occurred
multiple times. It has
become mottled
cyanotic, and flacid

the CD8+ T cells respond to peptides
presented by MHC Class I molecules

CD8+ T cells kill via two
mechanisms:
-perforin-granzyme-dependent
killing
-Fas-Fas ligand-dependent killing
h: Compare and Contrast Processes and Mechanisms of hyperacute, acute, and chronic
transplant rejection and recognize lab cases. P. 121-124 in text; also refer to IP Lab
** See Lab 6, Cases Kidney: Acute and Chronic Transplant Rejection
i. P. 126 – Understand concept of graft-versus-host reaction and which conditions
where it is most likely to occur.
When you have an organ transplant from a donor into a recipient, in almost all
cases that recipient is going to recognize that tissue as foreign. The only time that
doesn’t happen is if it is in an identical twin donor.
37
The reason it recognizes those tissues as foreign is due to the histocompatibility
Ag, or HLA. Rejection of foreign organs involves both cell-mediated immunity,
and Ab mediated mechanisms.
j. P. 125-144 Autoimmune diseases. Understand principles, pathogenisis, clinical
and pathologic features of these diseases and the lab cases where relevant: systemic
lupus erythematosus, rheumatoid arthritis, scleroderma, polyarteritis, nodosa, acute
rheumatic fever, multiple sclerosis, Graves’ Disease, Hashimoto’s thyroiditis,
Sjorgren’s Syndrome.
Autoimmune Diseases
1. Systemic lupus erythematosus (SLE)- SLE is a autoimmune, multisystem
disease of fluctuating manifestations and variable behavior. Clinically, it is
unpredictable, remitting/relapsing disease of acute onset that may involve any
organ of the body, but it principally affects the skin, kidneys, serosal
membranes, joints, and heart. The diagnostic criteria includes demonstrating
of the following: malar rash (fixed erythema), discoid rash (raised patches &
scaling), Photosensitivity, Oral Ulcers, Arthritis, Serositis (plueritic pain),
Renal Disorder, Neurologic Disorders (seizures, psychosis), Hematologic
disorders (anemias), Immunologic disorder (containing anti-DNA antibody, or
anti-sm nuclear antigen, or antiphospholipid antibodies). SLE affects 1/2500
person and more prevalent among women (9:1) esp. African-American
women. The fundamental defect in SLE is a failure to maintain self- tolerance.
Therefore, there is a generation of a wide array of autoantibodies that can
damage tissue either directly or in the form of immune complex deposits.
These antinuclear antibodies (ANAs) include: 1. antibodies to DNA, 2.
antibodies to histones, 3. antibodies to nonhistone proteins bound to RNA and
4. antibodies to nuclear antigens. Indirect immunofluroscence is used to test
for these ANAs. There is evidence supporting both genetic predisposition to
SLE and nongenetic factors including patients who take procainamide and
hydralazine. Sex hormones and ultra-violet exposure may also contribute.
Immunologic abnormalities particularly CD4+ helper T cell may attribute to
the pathogenesis. Also, most visceral lesions are mediated by type II
hypersensitivity and autoantibodies against blood cells via type II
hypersensitivity. Morphology: acute necrotizing vasculitis affecting small
arteries and arterioles, butterfly rash on face, joint deformity, enlarged spleen
with onion-skin lesions, neuropsychiatric symptoms, inflamed serosal
membranes, valvular lesions in the heart called Libman-Sacks endocarditis,
renal failure with various glomeruli defects. Finally most patients follow
course of remissions and relapses spanning years to decades.
2. Rheumatoid Arthritis (RA)- RA is a systemic, chronic inflammatory,
disease affecting multiple tissues but principally attacking the joints to
produce a nonsuppurative proliferative synovitis (histological) that frequently
progresses to destroy articular cartilage and underlying bone with resulting
disabling arthritis/joint inflammation. Morphology: affects the small joints of
the hands, feet, ankles, knees, wrists, elbows, and shoulders; pannus
38
3.
4.
5.
6.
(histological); and rheumatoid subcutaneous nodules. Pathogenesis: genetic
predisposition mediated by activation of helper T cells responding to some
possibly microbial arthritogenic agent.. These activated CD4+ cells produce
cytokines that activate macrophage in the joint space, releasing degradative
enzymes and other factors that perpetuate inflammation, and activate B cells,
resulting in the production of antibodies. RA patients may also have
weakness, malaise, and low grade fever. It first appears with aching and
stiffness of the joints. Some become stabilized or may regress, but most
pursue a chronic, remitting-relapsing course. After 10-20 years, the majority
of patients develop such deforming the destructive arthritis that surgical joint
replacement is required.
Scleroderma/Systemic Sclerosis (SS)- characterized by excessive fibrosis
through the body. Diffuse SS- widespread skin involvement and
limited/localized SS-confined to fingers and face aka CREST syndrome.
Fibroblast activation with excessive fibrosis is the hallmark of SS. The cause
is unknown, although it is attributed to abnormal activation of the immune
system and microvascular injury. Morphology: skin- fingers take on a tapered
claw like appearance with joint limitation; GI-tract- atrophy and collagen
replacement in the gut and esophagus, dysfunction of the lower sphincter;
pulmonary hypertension and renal abnormalities. Almost all patients develop
Raynaud phenomenon, a vascular disorder where the hands turn white when
exposed to cold, then blue, then red, as a presenting feature to CREST.
Lifespan may be normal in the absence of renal involvement.
Polyarteritis Nodosa- PN belongs to a group of diseases characterized by
necrotizing inflammation of the walls of blood vessels. The term
noninfectious necrotizing vasculitis differentiates PN from those conditions
attributable to direct vessel infection (i.e. absess) and serves to emphasize that
any type of vessel may be involved—arteries, veins, arterioles, or capillaries.
Sjogren Syndrome- characterized by dry eye and dry mouth resulting from
immune-mediated destruction of the lacrimal (thus, lack of tears) and salivary
glands. The ductal epithelial cells of the exocrine glands are the primary
target, and the is systemic B-cell hyperactivity. Most patients have
autoantibodies to the ribonucleoprotein (NRP) antigens SS-A (Ro) and SS-B
(La). Sjogren may lead to keratoconjunctivitis-drying of the corneal
epithelium with subsequent inflammation, erosion, and ulceration and
xerostomia- loss of salivary gland output leads to mucosal atrophy, with
inflammatory fissuring and ulceration. As most autoimmune diseases, 90% of
those infected are women.
Hasimoto Throditis- most common cause of hyperthyroidism, inflammatory
disorder of the thyroid. Although, HT is primarily caused by a defect in T
cells where CD4+ T cells induces formation of CD8+ and autoantibodies, it
involves both cellular and humoral responses where sensitized B cells secrete
inhibitory anti-TSH receptor antibodies that block the action of TSH.
Morphology includes an enlarged thyroid with microscopic views disclosing
widespread infiltration of the parenchyma by a mononuclear inflammatory
infiltrate containing small lymphocytes, plasma cells and well developed
39
germinal centers. The thyroid follicles of HT patients are lined with epithelial
cells(Hurthle/oxyphil cells) with abundant eosinophilic, granular cytoplasm.
7. Graves Disease- the most common cause of endogenous hyperthyroidism
characterized by a triad of manifestations: 1. Thyrotoxicosis caused by
hyperfunctional/ diffuse (hypertrophy and hyperplasia) enlargement of the
thyroid is present in all cases. 2. An opthalmopathy effect with resultant
exophtalmos in 40% of patients. 3. a localized, infiltrative dermopathy.
Graves disease mainly involves auto antigens against the TSH receptor.
8. Multiple Sclerosis is also a T cell-mediated disease where you have T cells
against myelin basic proteins in these patients. It is characterized by the
presence of plaques.
K. P. 144-147: Know pathologic abnormality, and cause for Bruton’s Disease,
Hashimoto’s thyroiditis, Sjorgren’s Syndrome.
Pathologic abnormality and cause for:
1. Bruton’s disease:
 X-linked agammaglobulinemia, so found primarily in males
 failure of pre-B cells to differentiate into B cells
 Normally not noticed until at least 6 months of age, because infant
receives immunity from mother up to that point
 Absence of gamma globulin in the blood
 No (or decreased) B cells in blood
 Underdeveloped lymphoid tissue
 No plasma cells
 Normal T cell-mediated responses
2. Common Variable Immunodeficiency:
 Heterogenous group of congenital or acquired disorders characterized by
hypogammaglobulinemia
 Impaired Ab response to infection
 Increased occurrence of infections
 Equally prevelant amongst males/females
 B cells present; plasma cells absent
3. IgA deficiency:
 Most common of all primary immunodeficiencies
 1/700 Caucasians affected
 Block in terminal differentiation of IgA-secreting B cells to plasma cells
 Most patients are asymptomatic
 Patients open to sinopulmonary infections and diarrhea
4. DiGeorge syndrome:
 Thymic hypoplasia
 Results from a congenital defect in thymic develepopment with deficient T
cell maturation
 T cells absent in lymphatics
 Infants susceptible to opportunistic infections
40
 Deletion affecting chromosome 22q11
5. SCID:
 Both humoral and cell-mediated immune response defecit
 In the 2 most common forms the thymus is hypoplastic and lymphoid
tissues atrophic
 Tx is bone marrow transplant
6. Wiscott-Aldrich syndrome:
 X-linked recessive disease
 Thrombocytopenia, eczema, vulnerability to recurrent infections, ending
in early death
 Only Tx is bone marrow transplant
 Patients susceptible to encapsulated pyogenic bacteria
7. Complement deficiency:
 Hereditary deficiency
 Problems depend on which complement molecule of lost
 (C3) increased susceptibility to infection with pyogenic bacteria
 (C1q, C2, C4) increased susceptibility to immune complex-mediated
disease (e.g. SLE)
 (C5-C8) recurrent neisserial (gonococcal, meningococcal) infections
L. P. 147-158 – AIDS/HIV – definitions, epidemiology – how AIDS is transmitted,
effects on the CD4 cells, and clinical manifestations.
I. DEFINITION: AIDS (acquired immunodeficiency syndrome):
a. retroviral dz caused by HIV (human immunodeficiency virus)
b. characterized by profound immunosuppression (especially cell-mediated
immunity) leading to opportunistic infections, secondary neoplasms, and
neurologic manifestations
c. STATISTICS:
i. 22 million have died since its discovery 20 yrs ago
ii. estimated 35 million globally have HIV (based on serologic data) –
that’s ~1/100
iii. 95% in developing countries; 50% in Africa
iv. AIDS = 5th leading cause of death in adults (25-44 yrs)
II. EPIDEMIOLOGY (based on American data):
a. 5 MAJOR GROUPS at risk for developing AIDS:
i. 46% Homosexual/bisexual males – largest group (14% also inject drugs)
– currently, on decline
ii. 25% Intravenous drug abusers – no history of homosexuality
iii. 11% heterosexual contacts of high risk groups (mostly drug abusers)
41
iv. 1% recipients of HIV-infected bld and bld components – platelets and
plasma
v. (1% of AIDS cases are children – not among major groups)
vi. <1% hemophiliacs – especially those that receive large amounts of
factor VIII/IX concentrates
vii. remaining 9-10% = unknown risk factors
b. TRANSMISSION of HIV – occurs under conditions that facilitate exchange of
bld or body fluids that contain virus or virus-infected cells;
i. 3 MAJOR ROUTES:
1. sexual contact – predominant mode worldwide (75% of cases)
a. virus present in semen, extracellular/intracellular in
mononuclear inflammatory cells, vaginal/cervical cells
(passed less efficiently to males)
b. enters recipients body through tears or abrasions in
mucosa
c. direct entry into bld vessels or uptake by dendritic cells
d. increased transmission with presence of other sexually
transmitted dzs
2. parenteral inoculation
a. ie) drug abusers, hemophiliacs receiving factors, blood
transfusions
b. sharing needles, syringes, paraphernalia
c. decreasing numbers through better screening of donated
bld
3. passage of virus from infected mothers to newborns
a. major cause of pediatrics AIDS
b. routes:
i. in utero, transplacental spread
ii. intrapartum – during delivery
iii. ingestion of HIV-contaminated breast milk
ii. HIV cannot be transmitted by casual personal contact; includes:
1. insect bites
2. touching, hugging, sharing household items
3. sharing facilities like swimming pools, saunas, toilets with HIVinfected ppl
iii. Very small but definite risk to health care workers
1. seroconversion – accidental needle stick inj = 0.3% per
accidental exposure
2. ie) transmission is extremely rare
III. PATHOGENESIS: - HIV infxn targets (1) immune system and (2) central nervous
system
42
a. Profound immunosuppression – especially cell-mediated immunity
b. Infection and subsequent loss of CD4+ Tcells (also targets macrophages and
dendritic cells that activate CD4+ Tcells)
c. CD4 is high-affinity receptor for HIV – but alone not enough
i. Initial binding to CD4
ii. conformational change exposing new adhesion sites for CCR5 and
CXCR4 coreceptors
iii. conformational change that allows cell-viral fusion
iv. HIV virus core enters cytoplasm of cell
v. viral genome undergoes reverse transcription forms cDNA
(complementary DNA)
1. quiescent cells – HIV proviral cDNA remains in cytoplasm
2. dividing cells – cDNA enters nucleus – integrated in host
genome
vi. latent infxn – nontranscribed for some period of time
vii. infxn = viral DNA transcription – extensive viral budding – cell death
1. since HIV-1 infects resting T cells – transcription initiated when
infected cell is activated (to Ag or cytokine) – T cell goes into
active mode
d. Needs coreceptors to ease cell entry – 2 cell surface chemokine receptors:
i. CCR5 – mostly on macrophages but also on monocytes
ii. CXCR4 – mostly on T cells (not on monocytes/macrophages)
e. HIV infxns characterized by profound loss of CD4+ cells from peripheral blood
i. initial loss is at a low rate (early in dz) = helper Tcells
ii. HIV colonizes lymphoid organs, infects T cell, macrophages, dendritic
cells
iii. Immune system is able to replace dying T cells initially
iv. But soon the system is overcome by BOTH increased DEATH and
reduced production
1. loss of immune precursors of CD4+ Tcells
2. fusion of infected and uninfected cells = formation of synctia
(giant cells)
3. uninfected CD4+ Tcells bind to viral glycoprotein – get Ag
activation through T cell receptor – leads to aberrant signaling
and apoptosis
4. cytotoxic CD8+ Tcell killing of infected CD4+ Tcells
5. ALL lead to inversion of CD4- CD8 ratio
v. Also get qualitative defects in Tcell function even in asymptomatic HIVinfected person
43
1. reduced Ag-induced Tcell proliferation, impaired TH1 cytokine
production, abnormal intracellular signaling
2. Reduction in CD4+ levels results in inability of peripheral
blood Tcells to be activated and challenge common Ags
vi. Levels of CD4+ gradually decrease due to productive infections –
which cannot be replenished; person begins to develop clinical
symptoms of AIDS
IV. CLINICAL FEATURES
a. Range from milk acute illness to severe disease
b. Each phase of infection has different features (see part N) – not explained in the
text
i. Acute Infection Syndrome - “flu-like” clinical illness
ii. Clinical “Latent” Disease - Lymphadenopathy and constitutional
symptoms
iii. Terminal phase (commonly known as AIDS) - development of
opportunistic infections
1. Fever
2. Weight loss
3. Diarrhea
4. Generalized lymphadenopathy
5. Secondary neoplasms
m.
Table 5-9: Immune system abnormalities in HIV/AIDS
Table 5-9 Major Abnormalities of Immune Function in AIDS
These are the FIVE major things you should know that go wrong in the immune
system in a patient that has AIDS.
1. They have a major reduction in their number of lymphocytes (Lymphopenia).
A.) This is mainly due to selective loss of CD4+ helper inducer T-cell subset.
B.) There is an inversion of the CD4/CD8 ratio.
2. They have a decreased T-Cell function In VivoA.) this makes the person susceptible to opportunistic infections
B.) there is an onset of tumors that can be benign or malignant (neoplasms)
C.) this results in a decreased Type IV delayed type hypersensitivity.
3. They have an Altered T-Cell Function in Vitro (in a test tube)A.) T cells of an AIDS patient have reduced proliferation response to
mitogens (substances that stimulate mitosis and lymphocyte
transformation), alloantigens ( antigens of the MHC), and soluble
antigens.
B.) The TC cells have a decreased specific cytotoxicity.
44
C.) The TH cells have decreased helper function for pokeweed mitogeninduced B-cell immunoglobulin production.
D.) There is decreased IL-2 and IFN-γ production.
4. Polyclonal B-Cell ActivationA.) There is an abnormally large amount of globulin proteins in the blood
plasma (hyperglobulinemia) as well as circulating immune complexes.
B.) There is an inability to mount de novo (newly synthesized metabolism
products) antibody response to a new antigen. (Basically, your body can’t
make antibodies to a new antigen that it has not seen before when you get
AIDS).
C.) Refractoriness to normal signals for B-cell activation in vitro. (B cell
activation is slowed.)
5. Altered Monocyte or Macrophage FunctionsA.) There is decreased chemotaxis.
B.) There is a decreased HLA class II antigen expression.
C.) There is a diminished capacity to present antigen to T-cells.
n. Understand 3 Stages of AIDS (p. 154)
1. Acute Phase- early phase- the initial response of an immunocompetent adult to HIV
infection. Typically a self-limited illness that develops in 50-70% of adults 3 to 6
weeks after infection. Characterized by nonspecific symptoms including sore throat,
myalgia, fever, rash, and sometimes aseptic meningitis. Characterized by high levels
of virus production, viremia, and widespread seeding of the peripheral lymphoid
tissues, typically w/ a reduction in CD4+ T cells. Soon a virus-specific immune
response develops, evidenced by seroconversion (usually w/in 3 to 17 weeks of
exposure) and by the development of virus-specific CD8+ cytotoxic T cells. As
viremia abates, CD4+ T cells return to nearly normal numbers. However, the
reduction in plasma virus does not signal the end of viral replication, which
continues w/in tissue macrophages and CD4+ T cells.
2. Chronic Phase-middle- a stage of relative containment of the virus. Immune system
is largely intact, but there is continued HIV replication lasting for several years.
Patients either are asymptomatic or develop persistent lymphadenopathy, and many
patients have “minor” opportunistic infections such as thrush or herpes zoster. Viral
replication in the lymphoid tissues continues unabated. The extensive viral turnover
is assoc. w/ loss of CD4+ cells. However, a large proportion of the CD4+ cells is
replenished. Thus, the decline of CD4+ cells in the peripheral blood is modest. After
an extended and variable period, host defense begins to wane, the number of CD4+
cells begins to decline, and the proportion of the surviving CD4+ cells infected with
HIV increases. Persistent lymphadenopathy w/ significant constitutional symptoms
45
(fever, rash, fatigue) reflects the onset of immune system decompensation, escalation
of viral replication, and the onset of the crisis phase.
3. Crisis Phase-final- Characterized by catastrophic breakdown of host defenses, a
marked increase in viremia, and clinical disease. Patients present w/ fever of more
than 1 month’s duration, fatigue, weight loss, and diarrhea; the CD4+ cell count is
reduced below 500cells/μL. After a variable interval, patients develop serious
opportunistic infections, secondary neoplasms, and/or neurologic manifestations (socalled AIDS-defining conditions), and the patient is said to have developed fullblown AIDS. Even in the usual AIDS-defining conditions are not manifest, CDC
guidelines define any HIV-infected individual w/ CD4+ cell counts less than or = to
200/μL as having AIDS.
o. Understand and be able to recognize major opportunistic infection agents in
HIV/AIDS (Table 5-10).


Most common (about 50%) infectious agent is the fungus Pneumocystis carinii.
o Causes pneumonia
o High risk in individuals with fewer than 200 CD4+ T cells/µL.
Other common organisms include
o Candida albicans – causes thrush
o Cytomegalovirus (CMV) - retinitis, GI disease
o Atypical Mycobacterial Disease
o Mycobacteria tuberculosis
o Cryptococcus neoformans- causes cryptococcosis, or cryptococcal
meningitis
o Toxoplasma gondii- responsible for causing toxoplasmosis, the second
most common secondary infection of the central nervous system.
o Diarrhea may be caused by protozoa, Salmonella, Shigella,
Cryptosporidium, and others.
p. Understand how HIV/AIDS is diagnosed clinically and confirmed by laboratory tests.
What should be done if a health care practitioner is exposed to body fluids where
HIV/AIDS infection may be a possibility?
I. The types of symptoms in primary infection are somewhat variable and not very reliable to
make the diagnosis. Only 50% of the infected people have symptoms. So it's possible to be
infected and never have any kind of illness associated with the infection . Others have this
flu like illness. Within about a month, you develop Abs that recognize HIV Ags. Measured
by two different kinds of assays.:
A. ELISA assay , or an enzyme immunoassay- measures Abs that bind to a groups of
proteins made by HIV.
B. confirmed by Western blot :
1. take viral proteins and run them out on an electrophoretic gel,
separate different proteins by size.
46
2.
Then you initially do the same assay of an ELISA assay, getting
g Abs to bind to these proteins, but you visualize it in different
bands. So if you have an Ab to some other Ag, maybe it cross
reacts with one of the proteins of HIV, but if you have Abs with
3 different molecular weights that bind to HIV, that's basically
highly unlikely that that's just a cross reaction, so it's used as a
confirmatory test that a person is infected with HIV.
Other TESTS:
1. Measure the amount of viral RNA that's in the plasma. Primarily used to monitor therapy
and see how rapidly progressing the disease is. It's not really a good way to measure if you're
infected except in the very early stages of infection . This is not used as a Diagnostic test.
2. measure P24 in the blood as an Ag - rarely used anymore. Viral load is a much more
sensitive measure of if there's a virus there in chronic phase.
3. CD4 count. CD4 count measure how close you are to the cliff. The higher the virus is
growing or the more virus in the blood, the faster the rate of decline CD4 cells. Normal CD4
1000 Cel / ul, Sick <350, AIDS <200..
4. Viral load, measures how fast you're moving toward the cliff.
II. If you've had a needle stick/exposure to HIV, post-exposure prophylactic with
antiretroviral drugs seem to work really well. You take antiretroviral drugs, like people that
are infected with the virus. If you start taking drugs within 2-3 days of being infected, the
chances are pretty good that you won't become seropositive even though you may have had a
few cells that got infected.
q. Life cycle of HIV-1: (FIG 5-38)
Primarily affecting cell-mediated immunity, is the hallmark of AIDS. This results
chiefly from infection and loss of CD4+ Tcells as well as impaired of macrophages and
dendritic cells.
The CD4+ molecule is a high-affinity receptor for HIV. Binding to CD4+ is not
sufficient for infection; the HIV envelope gp120 must bind to other cell surface
molecules (co receptors) to facilitate cell entry. CCR5 and CXCR4, two cell surface
chemokine receptor molecules, serve this role. HIV envelope gp120 (with a noncovalent
gp41) binds to the CD4 molecule which leads to a conformational change exposing a new
recognition site on gp120 for the CCR5 (mostly on Macrophage) and CXCR4 (mostly on
Tcells). The gp 41 undergoes a conformational change to insert a gp41 peptide into the
target membrane to facilitate the cell-viral fusion. After the fusion the virus core
containing the HIV genome enters the cytoplasm of the cell. The co-receptors CCR5 and
CXCR4 are critical components of the HIV infection process. (Ex.. individual with
defective CCR5 are relatively resistant to developing AIDS)
Once internalized, the viral genome undergoes reverse transcription leading to
formation of cDNA (c=complementary). In quiescent Tcells, HIV proviral cDNA can
remain in the cytoplasm in a linear episomal form. In a dividing Tcell, the cDNA enter
the nucleus and becomes integrated into the host genome. This step is probably a rate
limiting step in most kinds of cells. After the integration, the provirus can be latent for
47
months ( which mean it remains nontranscribed). Otherwise, the proviral DNA is
transcribed to form complete viral particles that bud from the cell membrane.
Once the virus is integrated however, there’s immediate viral activation. You get
viral RNA which is processed in several different ways. You produce a massive amount
of viral proteins, then like most viruses, the proteins formed by the virus basically selfassemble. They recognize each other and capture a piece of RNA to encapsulate it into a
virion, and then it buds off the cell surface. And once it’s outside the cell, an enzyme
that’s present in the virus has to proteolyze a thicker protein that’s in the capsule. And
that causes a conformational change in the virus that allows that virus to infect another
cell.
Infections associated with extensive viral budding lead to cell death. . HIV-1 can
infect resting Tcells, the initiation of proviral DNA transcription occurs only when the
infected cell is activated by and exposure to antigens or cytokines. Physiologic stimuli
that promote activation and growth of normal Tcells promote the death of HIV-infected
Tcells.
r. 158-164 – Amyloidosis – definition, appearance, diseases associations, and how to
detect it.
Definition – A variety of proteinaceous materials that are abnormally deposited in
tissue interstitium in a spectrum of clinical disorders.
Classification of amyloids:
1. Primary Amyloidosis – systemic, AL type, associated with some
immunocyte dyscrasia – most common form, 5-15% of patients often have
Multiple Myeloma (a malignant neoplasm of plasma cells) – most secrete
free light chains in serum or urine
2. Secondary Amyloidosis – AA type, systemic, occurs as a complication of
an underlying chronic inflammatory or tissue destructive process –
Rheumatoid arthritis, untreated TB, chronic osteomyelitis, IBD
Appearance – Diagnosis is based on histologic identification in a biopsy
specimen. It appears as an “amorphous, eosinophilic, hyaline extracellular
substance; with progressive accumulation, it encroaches on and produces pressure
atrophy of adjacent cells.” They are composed largely of non-branching fibrils
and proteoglycans.
*There are 3 main amyloid proteins:
1. Amyloid Light Chain (AL) – most prominent, contains Ig light chain –
deposition is associated with some form of monoclonal B cell proliferation
2. Amyloid Associated (AA) – non-Ig, serum amyloid-associated protein made
in the liver – typically deposited in the setting of chronic inflammatory
states
3. Beta-amyloid (AB) – found in the cerebral lesion of Alzheimer Dz
48
Disease associations – Amyloidosis should not be considered a single disease;
rather, it is a group of diseases that share in common the deposition of similar
appearing proteins. They can be systemic or localized.
Detection – Biopsy followed by Congo red staining is the most important tool in
the diagnosis of amyloidosis. Get a pink or red color of the tissue deposits with
light microscopy. Also get an “apple green” birefringence with polarizing
microscopy.
* See Figures 5-46 and 5-47 in book (pgs162-163)
49